Download as pdf or txt
Download as pdf or txt
You are on page 1of 38

CHAPTER 3

Chapter Overview
Section 1  discusses scalar and vector
quantities and graphical vector addition.
Section 2  explains the use of the
Pythagorean theorem and trigonometric
functions to find resultant vectors and
vector components.
Section 3  explores projectile motion,
neglecting air resistance.
Section 4  describes relative motion in
terms of vector operations.

About the Image


This fountain is located in Bayfront Park
in Sarasota, Florida. Point out that each
drop of water emerging from the
fountain moves as a projectile and obeys
the rules for projectile motion described
in this chapter. Likewise, the dolphins
move as projectiles as they sail through
the air.

Without air resistance, any object that


is thrown or launched into the air and

©Wendell Metzen/Index Stock Imagery/Photolibrary


that is subject to gravitational force
will follow a parabolic path. The water
droplets in this fountain are one
example. The velocity of any object in
two-dimensional motion—such as
one of these water droplets—can
be separated into horizontal and
vertical components, as shown in
the diagram.

Lab Preview
78

The following investigations support the DEMONSTRATIONS


concepts
Untitled-23 78 presented in this chapter. Vector Addition 5/9/2011 6:36:57 AM

Labs Air Resistance


Vector Treasure Hunt
Velocity of a Projectile
Projectile Motion (Probeware)
Projectile Motion (STEM)

QuickLab
Projectile Motion

78  Chapter 3
CHAPTER 3

Two-
SECTION 1
Introduction to
Vectors
SECTION 2 Focus and Motivate 

Dimensional
Vector Operations
SECTION 3 Activate Prior
Projectile Motion Knowledge
SECTION 4
Knowledge to Review

Motion and
Relative Motion • Displacement is a change in location
relative to a reference point.
• Velocity includes speed and direction.

Vectors
Why It Matters • Acceleration is the rate of change of
In this chapter, you will
velocity with respect to time. It has
use vectors to analyze both magnitude and direction.
two-dimensional motion
and to solve problems in Items to Probe
which objects are
• Displacement versus distance:  Have
projected into the air.
students decide displacement and
distance values for scenarios listed on
the board, such as round trips versus
one-way trips or a walk from one place
to another in the classroom.
• Acceleration and velocity:  Ask
students to describe the effects of
the four possible combinations of
acceleration and velocity in one
dimension: a > 0, v > 0; a < 0, v > 0;
a > 0, v < 0; a < 0, v < 0.

ONLINE Physics
HMDScience.com

ONLINE LABS
Vector Treasure Hunt
PREMIUM
Velocity of a Projectile CONTENT

Projectile Motion Physics


HMDScience.com

Projectile Motion

Why It Matters 79

Connecting to Art can be scaled up or down without changing


Untitled-23 79
In physics, a vector includes the direction of a resolution or pixelation. 5/9/2011 6:37:06 AM

quantity and its magnitude. In graphic design, Invite students who have knowledge of
vector graphics are images defined by computer-based art to discuss their experi-
mathematical equations rather than points in ences with vector graphics and bitmap
space. Vector graphics are drawn by comput- graphics. What are the differences between
ers following mathematical instructions. These these types of images? How can a vector
vector statements define where each line graphic be rendered as a bitmap or raster
should appear in an image. Because vector image, and what happens when this occurs?
graphics are rendered by actively drawn lines, How does the term vector graphics relate to
rather than by points mapped in space, they the use of the term vector in physics?

Two-Dimensional Motion and Vectors  79


SECTION  1 SECTION 1

 Plan and Prepare


Objectives
Distinguish between a scalar Introduction to
Vectors
and a vector.

Preview Vocabulary Add and subtract vectors by


using the graphical method.
Latin Word Origins  The word vector Key Terms
comes from the Latin word for carrier Multiply and divide vectors
scalar vector resultant
by scalars.
or conveyor. In biology and medicine,
Scalars and Vectors
the term is used to describe organisms
In the chapter “Motion in One Dimension,” our discussion of motion was
such as mosquitoes, fleas, and ticks that limited to two directions, forward and backward. Mathematically, we
carry and spread disease. In physics, this described these directions of motion with a positive or negative sign. That
term is used to visually represent method works only for motion in a straight line. This chapter explains a
method of describing the motion of objects that do not travel along a
quantities that have secondary charac- straight line.
teristics, such as direction.
Vectors indicate direction; scalars do not.
Each of the physical quantities encountered in this book can be catego-
 Teach scalar a physical quantity that has
magnitude but no direction
rized as either a scalar quantity or a vector quantity. A scalar is a quantity
that has magnitude but no direction. Examples of scalar quantities are
vector a physical quantity that has speed, volume, and the number of pages in this textbook. A vector is a

Teaching Tip
both magnitude and direction physical quantity that has both direction and magnitude.
Displacement is an example of a vector quantity. An airline pilot
Carry a paper cutout of an arrow with planning a trip must know exactly how far and which way to fly. Velocity is
you to class and use it as a tool to also a vector quantity. If we wish to describe the velocity of a bird, we must
specify both its speed (say, 3.5 m/s) and the direction in which the bird is
demonstrate the direction of vectors. flying (say, northeast). Another example of a vector quantity is acceleration.

The Language of Vectors are represented by boldface symbols.


Physics In physics, quantities are often represented by symbols, such as t for time.
To help you keep track of which symbols represent vector quantities and
FIGURE 1.1
Establish a convention for distinguishing which are used to indicate scalar quantities, this book will use boldface
vectors from scalars for use in your Length of Vector Arrows type to indicate vector quantities. Scalar quantities will be in italics. For
The lengths of the vector arrows example, the speed of a bird is written as v = 3.5 m/s. But a velocity,
classroom. Using an arrow above the represent the magnitudes of these which includes a direction, is written as v = 3.5 m/s to the northeast.
variable is a good substitute for bold two soccer players’ velocities. When writing a vector on your paper, you can distinguish it from a scalar
type, which would be hard to reproduce by drawing an arrow above the abbreviation for a quantity, such as
→v = 3.5 m/s to the northeast.
on a chalkboard. Make sure to be
One way to keep track of vectors and their directions is to use dia-
consistent when using this convention.
grams. In diagrams, vectors are shown as arrows that point in the direc-
For instance, when you use an arrow tion of the vector. The length of a vector arrow in a diagram is propor-
above the symbol to designate a vector, tional to the vector’s magnitude. For example, in Figure 1.1 the arrows
be sure to signify the direction of the represent the velocities of the two soccer players running toward the
soccer ball.
vector as well as its magnitude and units.

TEACH FROM VISUALS Differentiated


80 Chapter 3
Instruction

FIGURE 1.1  Tell students that the arrows Inclusion different. Ask students to imagine other
Visual and kinesthetic learners will benefit from a situations where speed is the same but
in the figure represent velocity, that is, Untitled-306 80 5/6/2011 11:57:42 AM

demonstration of scalar versus vector quantities. direction is different. For example, two cars
they represent both speed and
Invite two students to walk across the classroom traveling the same stretch of highway in
direction.
at the same speed but in opposite directions. As different directions.
Ask  Why do we consider velocity
instead of speed in order to decide a class, discuss the motion of the two students.
which player reaches the ball first? Is their speed the same? Yes. Is their motion the
same? No. How is it different? They were going
Answer: Direction is important because
in different directions.
a player may run very fast but not
toward the ball. The scalar quantity (speed) is the same for
these two students, but their vector is

80  Chapter 3
A resultant vector represents the sum of two or more vectors. Did YOU Know?
When adding vectors, you must make certain that they have the same The word vector is also used by airline
units and describe similar quantities. For example, it would be meaning-
less to add a velocity vector to a displacement vector because they
pilots and navigators. In this context,
a vector is the particular path followed
Demonstration
describe different physical quantities. Similarly, it would be meaningless, or to be followed, given as a compass
as well as incorrect, to add two displacement vectors that are not ex- heading. Vector Addition
pressed in the same units. For example, you cannot add meters and feet Purpose  Preview force as a vector
together. quantity to demonstrate vector
The chapter “Motion in One Dimension” covered vector addition and addition.
subtraction in one dimension. Think back to the example of the gecko
that ran up a tree from a 20 cm marker to an 80 cm marker. Then the Materials  large, empty box
gecko reversed direction and ran back to the 50 cm marker. Because the
two parts of this displacement are each vectors, they can be added
Procedure  Ask for a student volunteer.
together to give a total displacement of 30 cm. The answer found by Explain to the class that you will push
adding two vectors in this way is called the resultant. resultant a vector that represents the the box in one direction while the
sum of two or more vectors
student will push the box in a second,
Vectors can be added graphically. perpendicular direction at the same
Consider a student walking 1600 m to a friend’s house and then 1600 m to time. Ask students to predict the
school, as shown in Figure 1.2. The student’s total displacement during his
walk to school is in a direction from his house to the school, as shown by
motion of the box.
the dotted line. This direct path is the vector sum of the student’s displace- Have the student volunteer take a
ment from his house to his friend’s house and his displacement from the
friend’s house to school. How can this resultant displacement be found?
practice push alone and emphasize that
One way to find the magnitude and direction of the student’s total
the box moves in the direction of the
displacement is to draw the situation to scale on paper. Use a reasonable push. Take a practice push by yourself
scale, such as 50 m on land equals 1 cm on paper. First draw the vector on the box and again emphasize that the
representing the student’s displacement from his house to his friend’s
house, giving the proper direction and scaled magnitude. Then draw the
box moves in the direction of the push.
vector representing his walk to the school, starting with the tail at the Return the box to its original location.
head of the first vector. Again give its scaled magnitude and the right On the count of three, both you and
direction.
the student push the box in a direction
The magnitude of the resultant vector can FIGURE 1.2
then be determined by using a ruler. Measure
perpendicular to each other. Ask a
the length of the vector pointing from the tail Graphical Method of Vector Addition A student walks from student to explain why the box moved
his house to his friend’s house (a), then from his friend’s house to the
of the first vector to the head of the second along the diagonal. The box moved in a
vector. The length of that vector can then be school (b). The student’s resultant displacement (c) can be found by
using a ruler and a protractor. direction between the directions of the
multiplied by 50 (or whatever scale you have
chosen) to get the actual magnitude of the two pushes.
student’s total displacement in meters.
The direction of the resultant vector may be
determined by using a protractor to measure (b)
the angle between the resultant and the first
vector or between the resultant and any (c)
chosen reference line.
(a)

Problem Solving Two-Dimensional Motion and Vectors 81

Reality Check box look like? Show a stick figure pushing the
Untitled-306 81
Drawing a picture, as in Figure 1.2, is a good box from another side, with an arrow 5/6/2011
in the11:57:43 AM
way for students to confirm that they under- direction of that push. Finally, what do the
stand the meaning of the vectors in a given vectors look like when you add them together
situation. For example, after completing the –that is, when both people are pushing on the
demonstration, invite a student to draw each box at the same time? Drawing should show
situation on the board, along with its vector. two stick figures pushing at the same time
from perpendicular directions, with one arrow
What should one student pushing alone
showing the resultant vector—which is the
look like? Student should draw a stick figure
actual motion of the box.
pushing a box and an arrow in the direction of
the push. What does the teacher pushing the

Two-Dimensional Motion and Vectors  81


Properties of Vectors

 Teach continued
Now consider a case in which two or more vectors act at the same point.
When this occurs, it is possible to find a resultant vector that has the same
net effect as the combination of the individual vectors. Imagine looking
down from the second level of an airport at a toy car moving at 0.80 m/s
Misconception Alert! across a walkway that moves at 1.5 m/s. How can you determine what the
car’s resultant velocity will look like from your view point?
Students may have difficulty visualizing
the movement of the toy car on the
FIGURE 1.3 Vectors can be moved parallel to themselves in a diagram.
walkway. Illustrate the situation on the
Note that the car’s resultant velocity while moving from one side of the
board, and emphasize that the motion Triangle Method of Addition
The resultant velocity (a) of a toy
walkway to the other will be the combination of two independent motions.
represented in Figure 1.3 is drawn as car moving at a velocity of 0.80 m/s Thus, the moving car can be thought of as traveling first at 0.80 m/s across
viewed by a stationary observer above (b) across a moving walkway with a the walkway and then at 1.5 m/s down the walkway. In this way, we can
velocity of 1.5 m/s (c) can be found draw a given vector anywhere in the diagram as long as the vector is parallel
the walkway. Frame of reference will be to its previous alignment (so that it still points in the same direction).
using a ruler and a protractor.
covered later in the chapter. Thus, you can draw one vector with its tail starting at the tip of the
vwalkway = 1.5 m/s
other as long as the size and direction of each vector do not change. This
(c) process is illustrated in Figure 1.3. Although both vectors act on the car at

The Language of the same point, the horizontal vector has been moved up so that its tail
vcar = 0.80 m/s

(a) begins at the tip of the vertical vector. The resultant vector can then be
(b)
Physics vresultant
drawn from the tail of the first vector to the tip of the last vector. This
method is known as the triangle (or polygon) method of addition.
The triangle method of vector addition Again, the magnitude of the resultant vector can be measured using a
Car
is also called the polygon method, the ruler, and the angle can be measured with a protractor. In the next section,
we will develop a technique for adding vectors that is less time-consuming
head-to-tail method, or the tip-to-tail
because it involves a calculator instead of a ruler and protractor.
method.
FIGURE 1.4 Vectors can be added in any order.
When two or more vectors are added, the sum is
TEACH FROM VISUALS Commutative Property of Vectors A marathon runner’s
displacement, d, will be the same regardless of whether the runner takes
independent of the order of the addition. This
path (a) or (b) because the vectors can be added in any order. idea is demonstrated by a runner practicing for a
FIGURE 1.4  Reinforce your students’ marathon along city streets, as represented in
understanding of the difference Figure 1.4. The runner executes the same four
displacements in each case, but the order is
between distance and displacement by d d
different. Regardless of which path the runner
pointing out that if the runner starts takes, the runner will have the same total
and ends at the positions shown in displacement, expressed as d. Similarly, the
vector sum of two or more vectors is the same
Figure 1.4, the runner’s displacement will regardless of the order in which the vectors are
be the same regardless of the path the (a) (b)
added, provided that the magnitude and direc-
runner chooses to follow. tion of each vector remain the same.

Ask  How can the runner increase


To subtract a vector, add its opposite.
distance but begin and end at the same
Vector subtraction makes use of the definition of the negative of a vector.
points? The negative of a vector is defined as a vector with the same magnitude as
Answer: The runner can take a different, the original vector but opposite in direction. For instance, the negative of
the velocity of a car traveling 30 m/s to the west is −30 m/s to the west,
longer path but will have to double back
somewhere along the path to end up at
Problem
82
Solving
Chapter 3
the same point.
Reality Check A. A paper airplane moves at a constant
Students will better understand the
Untitled-306 82 velocity of 10 m/s. What is its resultant 5/6/2011 11:57:44 AM

commutative property of vector addition if velocity if it moves with a parallel wind


they work several examples on paper with a gust of 19 m/s?
ruler and protractor. Provide students with the B. A stunt man tries to run backward on a
following examples and encourage them to treadmill. The stuntman’s constant
illustrate each situation before drawing the velocity is 8 m/s and the treadmill is
vectors. The actual answer is not important at moving at a velocity of 9 m/s. What is
this point. Instead, emphasize that students the resultant velocity?
should be able to set up each problem and
draw the vectors.

82  Chapter 3
or +30 m/s to the east. Thus, adding a vector to its negative vector gives
zero. When subtracting vectors in two dimensions, first draw the negative
of the vector to be subtracted. Then add that negative vector to the other Teaching Tip
vector by using the triangle method of addition.
Some students may need further
explanation and visual examples of the
Multiplying or dividing vectors by scalars results in vectors.
negative of a vector. Give several
There are mathematical operations in which vectors can multiply other
vectors, but they are not needed in this book. This book does, however, examples of vectors in different
make use of vectors multiplied by scalars, with a vector as the result. For directions and then show their negative
example, if a cab driver obeys a customer who tells him to go twice as fast,
vectors.
that cab’s original velocity vector, vcab, is multiplied by the scalar num-
ber 2. The result, written 2vcab, is a vector with a magnitude twice that of
the original vector and pointing in the same direction.
On the other hand, if another cab driver is told to go twice as fast in
the opposite direction, this is the same as multiplying by the scalar
Assess and Reteach 
number −2. The result is a vector with a magnitude two times the initial
velocity but pointing in the opposite direction, written as −2vcab. Assess  Use the Formative Assessment
on this page to evaluate student
mastery of the section.
SECTION 1 FORMATIVE ASSESSMENT Reteach  For students who need
additional instruction, download the
Reviewing Main Ideas Section Study Guide.
1. Which of the following quantities are scalars, and which are vectors?
a. the acceleration of a plane as it takes off
Response to Intervention  To reassess
b. the number of passengers on the plane students’ mastery, use the Section Quiz,
c. the duration of the flight available to print or to take directly
d. the displacement of the flight online at HMDScience.com.
e. the amount of fuel required for the flight
2. A roller coaster moves 85 m horizontally, then travels 45 m at an angle
of 30.0° above the horizontal. What is its displacement from its starting
point? Use graphical techniques.
3. A novice pilot sets a plane’s controls, thinking the plane will fly at
2.50 × 102 km/h to the north. If the wind blows at 75 km/h toward
the southeast, what is the plane’s resultant velocity? Use graphical
techniques.
4. While flying over the Grand Canyon, the pilot slows the plane down to
one-half the velocity in item 3. If the wind’s velocity is still 75 km/h
toward the southeast, what will the plane’s new resultant velocity be?
Use graphical techniques.

Critical Thinking
5. The water used in many fountains is recycled. For instance, a single water
particle in a fountain travels through an 85 m system and then returns
to the same point. What is the displacement of this water particle during
one cycle?

Answers to Section Assessment Two-Dimensional Motion and Vectors 83

1. a. vector
ntitled-306 83 b. scalar 5/6/2011 11:57:44 AM

c. scalar
d. vector
e. scalar
2. 126 m at (1.0 × 101)° above the horizontal
3. 204 km/h at 75° north of east
4. 89 km/h at 54° north of east
5. zero

Two-Dimensional Motion and Vectors  83


SECTION  2 SECTION 2

 Plan and Prepare


Objectives
Identify appropriate coordinate
systems for solving problems
Vector Operations
Preview Vocabulary with vectors.

Apply the Pythagorean theorem


Key Term
components of a vector
Scientific Meaning  The word and tangent function to
simultaneous is used for phenomena calculate the magnitude and
Coordinate Systems in Two Dimensions
that occur together at the same time. direction of a resultant vector.
In the chapter “Motion in One Dimension,” the motion of a gecko
Ask students to list some simultaneous Resolve vectors into climbing a tree was described as motion along the y-axis. The direction
phenomena, such as the pressure of a components using the sine and of the displacement of the gecko was denoted by a positive or negative
cosine functions. sign. The displacement of the gecko can now be described by an arrow
confined gas decreasing while its volume
pointing along the y-axis, as shown in Figure 2.1. A more versatile system
is increasing. Add vectors that are not
for diagramming the motion of an object, however, employs vectors and
perpendicular.
the use of both the x- and y-axes simultaneously.
The addition of another axis helps describe motion in two dimensions
 Teach FIGURE 2.1
and simplifies analysis of motion in one dimension. For example, two
methods can be used to describe the motion of a jet moving at 300 m/s to
Using a Coordinate System the northeast. In one approach, the coordinate system can be turned so

Teaching Tip A gecko’s displacement while climbing that the plane is depicted as moving along the y-axis, as in Figure 2.2(a).
a tree can be represented by an arrow The jet’s motion also can be depicted on a two-dimensional coordinate
system whose axes point north and east, as shown in Figure 2.2(b).
Review the sign conventions for pointing along the y-axis.
One problem with the first method is that the axis must be turned
coordinate systems that were estab-
again if the direction of the plane changes. Another problem is that the
lished in the chapter “Motion in One first method provides no way to deal with a second airplane that is not
Dimension.” Movements to the right traveling in the same direction as the first airplane. Thus, axes are often
designated using fixed directions. For example, in Figure 2.2(b), the
along the x-axis and upward along the
positive y-axis points north and the positive x-axis points east. Similarly,
y-axis are considered positive, and when you analyze the motion of objects thrown into the air, orienting the
movements to the left along the x-axis y-axis parallel to the vertical direction simplifies problem solving.
and downward along the y-axis are
Tips and Tricks
considered negative. There are no firm rules for applying coordinate systems to situations involving vectors.
As long as you are consistent, the final answer will be correct regardless of the system
you choose. Perhaps your best choice for orienting axes is the approach that makes
solving the problem easiest for you.

TEACH FROM VISUALS

FIGURE 2.2  Point out the two very FIGURE 2.2 N y


different choices for coordinate axes. Two Different Coordinate
W E
Ask  Which set of axes will give the Systems A plane traveling northeast at
a velocity of 300 m/s can be represented
correct answer? as either (a) moving along a y-axis chosen S

Answer: Either set of axes must give the to point to the northeast or (b) moving at
an angle of 45° to both the x- and y-axes, v = 300 m/s at 45˚
same result. which line up with west-east and south- v = 300 m/s northeast x
north, respectively.
(a) (b)

Problem
84
Solving
Chapter 3

Take It Further Then ask, how would the situation be


For Figure 2.2, ask students how the plane’s
Untitled-241 84 different with a tailwind of 10 m/s from the 5/4/2011 2:50:58 PM

motion would be changed if there is a southwest? The plane’s velocity would be


headwind of 10 m/s from the northeast. The increased to 310 m/s. What would be the total
plane’s velocity would be reduced to 290 m/s difference in velocity between a headwind of
northwest. Invite a student to draw the 10 m/s and a tailwind of 10 m/s? 20 m/s
vectors for this situation on the board, and to
show classmates how to solve this problem
both visually and mathematically.

84  Chapter 3
Determining Resultant Magnitude and Direction
Earlier, we found the magnitude and direction of a resultant graphically.
However, this approach is time-consuming, and the accuracy of the answer
Misconception Alert!
depends on how carefully the diagram is drawn and measured. A simpler Students often try to apply the
method uses the Pythagorean theorem and the tangent function. Pythagorean theorem to triangles that
do not contain a right angle. Point out
Use the Pythagorean theorem to find the magnitude of the resultant. FIGURE 2.3 that the Pythagorean theorem can be
Imagine a tourist climbing a pyramid in Egypt. The tourist knows the
A Triangle Inside of a Pyramid used only with a right triangle. Some
height and width of the pyramid and would like to know the distance Because the base and height of a pyramid
covered in a climb from the bottom to the top of the pyramid. Assume students may know the Law of Cosines,
are perpendicular, we can find a tourist’s
that the tourist climbs directly up the middle of one face. total displacement, d, if we know the which applies to all triangles. This law
As can be seen in Figure 2.3, the magnitude of the tourist’s vertical height, ∆y, and width, 2∆x, of the states that c2 = a2 + b2 − 2abcosθ. The
displacement, ∆y, is the height of the pyramid. The magnitude of the pyramid.
Law of Cosines can be used to calculate
horizontal displacement, ∆x, equals the distance from one edge of the
pyramid to the middle, or half the pyramid’s width. Notice that these one side of any triangle when the
two vectors are perpendicular and form a right triangle with the dis- ∆y opposite angle and the lengths of the
placement, d.
∆x d
other two sides are known. In this
As shown in Figure 2.4(a), the Pythagorean theorem states that for any
expression, c is the unknown side, θ is
right triangle, the square of the hypotenuse—the side opposite the right
angle—equals the sum of the squares of the other two sides, or legs. the angle opposite c, and a and b are
2∆x the two known sides. Some students
may attempt to use the Law of Cosines
Pythagorean Theorem for Right Triangles
c2 = a2 + b2
to add nonperpendicular vectors. This
approach will give the correct answer,
(length of hypotenuse)2 = (length of one leg)2 + (length of other leg)2 but it entails more computation and is
more prone to student error when more
In Figure 2.4(b), the Pythagorean theorem is applied to find the tourist’s than two vectors are to be added.
displacement. The square of the displacement is equal to the sum of the
square of the horizontal displacement and the square of the vertical
displacement. In this way, you can find out the magnitude of the dis-
placement, d. Teaching Tip
Point out that finding the resultant for
FIGURE 2.4 the pyramid is fairly simple because the
Using the Pythagorean Theorem height, half-width, and hypotenuse form
(a) The Pythagorean theorem can be applied to any right triangle. a right triangle. It is important to
(b) It can also be applied to find the magnitude of a resultant displacement. mention at this point that right triangles
will also allow students to find the x and
y components that are important for
d y
c
vector addition.
a

(a) b (b) x
c 2 = a2 + b 2 d2 = x2 + y2

Two-Dimensional Motion and Vectors 85

Deconstructing Problems
Untitled-241 85
Emphasize the importance of recognizing what 5/4/2011 2:50:59 PM

each symbol in an equation stands for. For


example, in d 2 = Δx2 + Δy2, the Δ is neither
a variable nor a coefficient. In fact, both Δx
and Δy are unique variables. That is, Δx or Δy
can be replaced with only one value. Challenge
students to investigate more about the
symbol Δ, delta, and what it stands for in
mathematics and physics.

Two-Dimensional Motion and Vectors  85


FIGURE 2.5 Use the tangent function to find the direction of the resultant.
In order to completely describe the tourist’s displacement, you must also
 Teach continued Using the Tangent
Function (a) The tangent function
can be applied to any right triangle,
know the direction of the tourist’s motion. Because ∆x, ∆y, and d form a
right triangle, as shown in Figure 2.5(b), the inverse tangent function can be
and (b) it can also be used to find the used to find the angle θ, which denotes the direction of the tourist’s
Classroom Practice direction of a resultant displacement. displacement.
For any right triangle, the tangent of an angle is defined as the ratio of
Finding Resultant Magnitude and the opposite and adjacent legs with respect to a specified acute angle of a
Hypotenuse
Direction  A plane travels from Houston, Opposite right triangle, as shown in Figure 2.5(a).
Texas, to Washington, D.C., which is (a) As shown below, the magnitude of the opposite leg divided by the
Adjacent
1540 km east and 1160 km north of magnitude of the adjacent leg equals the tangent of the angle.
opp
tan =
Houston. What is the total displacement adj

of the plane? Definition of the Tangent Function for Right Triangles

d opp opposite leg


Answer: 1930 km at 37.0° north of east y tan θ = _ tangent of angle = __
adj adjacent leg
A camper travels 4.5 km northeast and (b)
x
4.5 km northwest. What is the camper’s The inverse of the tangent function, which is shown below, gives
tan = the angle.
total displacement?
Answer: 6.4 km north = tan-1
θ = tan−1 _
( )
opp
adj

Teaching Tip Finding Resultant Magnitude and Direction

Explain that trigonometric functions Sample Problem A An archaeologist climbs the Great
Pyramid in Giza, Egypt. The pyramid’s height is 136 m and its
such as the one in the text, tan‒1, have a width is 2.30 × 102 m. What is the magnitude and the direction of
different role. This is not an exponent. the displacement of the archaeologist after she has climbed from
Instead, it is used to represent the the bottom of the pyramid to the top?

inverse of a trigonometric function. For ANALYZE Given: ∆y = 136 m 1 (width) = 115 m


∆x = _
example, the inverse of the functions 2
Unknown: d=? θ=?
sin x = 0.5 and cos 3a = 0.92 take on
Diagram: Choose the archaeologist’s starting position
the following forms: as the origin of the coordinate system.
x = sin‒1 0.5 = 30°
cos-1 0.92
3a = cos‒1 0.92 ⇒ a = ​ _  ​   = 7.7° y
3
d y = 136 m

x = 115 m x

Continued

Problem
86
Solving
Chapter 3

Take It Further
Give students directions for a treasure hunt
Untitled-241 86 5/4/2011 2:51:00 PM

with movements that are all perpendicular to


one another (i.e, move front/back and right/
left). Have a student follow the directions to
find a “treasure”. Then have the class resolve
the directions into two components and
calculate the resultant vector. Have a student
use the resultant vector to go directly from
the start position to the treasure.

86  Chapter 3
Finding Resultant Magnitude and Direction (continued)

PROBLEM guide A
PLAN Choose an equation or situation:
The Pythagorean theorem can be used to find the magnitude of the Use this guide to assign problems.
archaeologist’s displacement. The direction of the displacement can
be found by using the tangent function.
SE = Student Edition Textbook
d 2 = ∆x2 + ∆y2 PW = Sample Problem Set I (online)
∆y PB = Sample Problem Set II (online)
tan θ = _
∆x
Tips and Tricks Solving for:
Be sure your calculator is set
Rearrange the equations to isolate the unknowns: to calculate angles measured resultant SE Sample, 1–4;
in degrees. Some calculators Ch. Rvw.
d= √�����
∆x2 + ∆y2 have a button labeled “DRG”
that, when pressed, toggles 21–22, 23*
∆y
θ = tan-1 _
∆x( ) between degrees, radians,
and grads. PW 2, 4–5, 7*
PB Sample, 1–5
SOLVE Substitute the values into the equations and solve: component PW Sample, 1, 3*, 6*
d= √��������
(115 m)2 + (136 m)2 PB 6–10
d = 178 m *Challenging Problem

(
136 m
θ = tan-1 _
115 m )
θ = 49.8° Answers
CHECK YOUR Because d is the hypotenuse, the archaeologist’s displacement should Practice A
WORK be less than the sum of the height and half of the width. The angle is 1. a. 23 km
expected to be more than 45° because the height is greater than half of
the width. b. 17 km to the east
2. 45.6 m at 9.5° east of north
1. A truck driver is attempting to deliver some furniture. First, he travels 8 km east, 3. 15.7 m at 22° to the side of downfield
and then he turns around and travels 3 km west. Finally, he turns again and travels
12 km east to his destination. 4. 1.8 m at 49° below the horizontal
a. What distance has the driver traveled?
b. What is the driver’s total displacement?

2. While following the directions on a treasure map, a pirate walks 45.0 m north and
then turns and walks 7.5 m east. What single straight-line displacement could the
pirate have taken to reach the treasure?

3. Emily passes a soccer ball 6.0 m directly across the field to Kara. Kara then kicks
the ball 14.5 m directly down the field to Luisa. What is the total displacement of
the ball as it travels between Emily and Luisa?

4. A hummingbird, 3.4 m above the ground, flies 1.2 m along a straight path. Upon
spotting a flower below, the hummingbird drops directly downward 1.4 m to hover
in front of the flower. What is the hummingbird’s total displacement?

Two-Dimensional Motion and Vectors 87

Alternative Approaches
ntitled-241 87
To find an angle in a right triangle, you can 5/4/2011 2:51:01 PM

apply different trigonometric functions, such


as the following.

( )
θ = sin‒1 ​ _
∆y
( )136
​   ​   ​= sin‒1 ​ ​ _  ​  ​= 49.8°
d 178

Two-Dimensional Motion and Vectors  87


Resolving Vectors into Components
In the pyramid example, the horizontal and vertical parts that add up to
 Teach continued components of a vector the
projections of a vector along the axes
of a coordinate system
give the tourist’s actual displacement are called components. The x compo-
nent is parallel to the x-axis. The y component is parallel to the y-axis. Any
vector can be completely described by a set of perpendicular components.
Teaching Tip In this textbook, components of vectors are shown as outlined, open
In mathematics, the components of a arrows. Components have arrowheads to indicate their direction.
Components are scalars (numbers), but they are signed numbers. The
vector are called projections. The x
direction is important to determine their sign in a coordinate system.
component is the projection of the You can often describe an object’s motion more conveniently by
vector along the x-axis, and the y breaking a single vector into two components, or resolving the vector.
component is the projection of the Resolving a vector allows you to analyze the motion in each direction.
vector along the y-axis. FIGURE 2.6 This point is illustrated by examining a scene on the set of an action
movie. For this scene, a plane travels at 95 km/h at an angle of 20° relative
Diagramming a Movie Scene to the ground. Filming the plane from below, a camera team travels in a
A truck carrying a film crew must be driven truck directly beneath the plane at all times, as shown in Figure 2.6.
Misconception Alert! at the correct velocity to enable the crew
to film the underside of a plane. The plane
To find the velocity that the truck must maintain to stay beneath the
plane, we must know the horizontal component of the plane’s velocity.
Because of the prominence of angles flies at 95 km/h at an angle of 20° relative
Once more, the key to solving the problem is to recognize that a right
to the ground.
measured from the x-axis, students may triangle can be drawn using the plane’s velocity and its x and y compo-
develop the misconception that the x v plane nents. The situation can then be analyzed using trigonometry.
component of a vector is always The sine and cosine functions are defined in terms of the lengths of
the sides of such right triangles. The sine of an angle is the ratio of the leg
calculated using the cosine function. opposite that angle to the hypotenuse.
This misconception may be corrected by 20˚
using examples on the board in which vtruck
Definition of the Sine Function for Right Triangles
the angles are measured from the y-axis.
sin θ = _
opp
sine of an angle = __
opposite leg
hyp hypotenuse

In Figure 2.7, the leg opposite the 20° angle represents the y compo-
nent, vy, which describes the vertical speed of the airplane. The hypot-
enuse, vplane, is the resultant vector that describes the airplane’s total
velocity.
The cosine of an angle is the ratio between the leg adjacent to that
angle and the hypotenuse.
FIGURE 2.7

Using Vector Components


Definition of the Cosine Function for Right Triangles
To stay beneath the biplane, the truck
must be driven with a velocity equal to the
x component (vx ) of the biplane’s velocity. cos θ = _
adj
cosine of an angle = __
adjacent leg
hyp hypotenuse

plane = 95 km/h In Figure 2.7, the adjacent leg represents the x component, vx, which
describes the airplane’s horizontal speed. This x component equals the
20˚
speed required of the truck to remain beneath the plane. Thus, the truck
must maintain a speed of vx = (cos 20°)(95 km/h) = 90 km/h.

Differentiated
88 Chapter 3
Instruction
Below level
Students who need a refresher on trigonometry
Untitled-241 88 5/4/2011 2:51:02 PM

can be directed to Appendix A, which includes


a more detailed discussion of the sine, cosine,
and tangent functions, as well as the
Pythagorean theorem.

88  Chapter 3
PREMIUM CONTENT

Resolving Vectors Interactive Demo

Sample Problem B Find the components of the velocity of a


HMDScience.com
Classroom Practice
helicopter traveling 95 km/h at an angle of 35° to the ground.
Resolving Vectors  An arrow is shot
ANALYZE Given: v = 95 km/h θ = 35° from a bow at an angle of 25° above
Unknown: vx = ? vy = ?
the horizontal with an initial speed of
= 95 km/h
45 m/s. Find the horizontal and vertical
Diagram: The most convenient coordinate system
is one with the x-axis directed along the components of the arrow’s initial
35˚
ground and the y-axis directed vertically. velocity.
Answer: 41 m/s, 19 m/s
The arrow strikes the target with a
PLAN Choose an equation or situation:
Because the axes are perpendicular, the sine and cosine functions can speed of 45 m/s at an angle of −25°
be used to find the components. with respect to the horizontal. Calculate
vy Tips and Tricks
sin θ = _ the horizontal and vertical components
v Don’t assume that the cosine
vx
function can always be used for of the arrow’s final velocity.
the x-component and the sine
cos θ = _
v function can always be used for Answer: 41 m/s, −19 m/s
the y-component. The correct
Rearrange the equations to isolate the unknowns: choice of function depends on
vy = v sin θ where the given angle is located.

vx = v cos θ
Instead, always check to see
which component is adjacent and PROBLEM guide B
which component is opposite to
the given angle. Use this guide to assign problems.
SE = Student Edition Textbook
SOLVE Substitute the values into the equations and solve: PW = Sample Problem Set I (online)
vy = (95 km/h) (sin 35°) PB = Sample Problem Set II (online)
vy = 54 km/h Solving for
one SE 1–2
vx = (95 km/h) (cos 35°)
component PW Sample, 1, 3
vx = 78 km/h
PB 1–4
both SE Sample, 3–4;
CHECK YOUR Because the components of the velocity form a right triangle with the components Ch. Rvw. 24–25
helicopter’s actual velocity, the components must satisfy the
ANSWER PW 2–8
Pythagorean theorem.
v 2 = vx2 + vy2 PB Sample, 5–10
(95)2 = (78)2 + (54)2 *Challenging Problem
9025 ≈ 9000
The slight difference is due to rounding.

Continued

Problem Solving Two-Dimensional Motion and Vectors 89

Alternative Approach
ntitled-241 89
For Step 4 (Check Your Answer) students can 5/4/2011 2:51:02 PM

apply the sine and cosine ratios as shown:


vy 54
​ v ​  ⇒ sin 35° = _
sin θ   = _ ​    ​ ⇒ 0.573 ≈ 0.568
95
v 78
cos θ = _ ​ vx ​  ⇒ cos 35° = _​    ​ ⇒ 0.819 ≈ 0.821
95

The slight difference is due to rounding.

Two-Dimensional Motion and Vectors  89


Resolving Vectors (continued)

 Teach continued
1. How fast must a truck travel to stay beneath an airplane that is moving 105 km/h

Answers
at an angle of 25° to the ground?

2. What is the magnitude of the vertical component of the velocity of the plane in item 1?
Practice B
1. 95 km/h 3. A truck drives up a hill with a 15° incline. If the truck has a constant speed of
22 m/s, what are the horizontal and vertical components of the truck’s velocity?
2. 44 km/h
4. What are the horizontal and vertical components of a cat’s displacement when the
3. 21 m/s, 5.7 m/s cat has climbed 5 m directly up a tree?
4. 0 m, 5 m

Adding Vectors That Are Not Perpendicular


Teaching Tip Until this point, the vector-addition problems concerned vectors that are
Problems involving vectors that are not perpendicular to one another. However, many objects move in one
direction and then turn at an angle before continuing their motion.
perpendicular use both vector addition
Suppose that a plane initially travels 5 km at an angle of 35° to the
and vector resolution. Because they act
ground, then climbs at only 10° relative to the ground for 22 km. How can
as a nice summary to the concepts of you determine the magnitude and direction for the vector denoting the
this section, you may want to do several total displacement of the plane?
examples involving this type of problem. Because the original displacement vectors do not form a right triangle,
you can not apply the tangent function or the Pythagorean theorem when
These problems require a methodical
adding the original two vectors.
approach to problem solving, which
Determining the magnitude and the direction of the resultant can be
should prove helpful to students while achieved by resolving each of the plane’s displacement vectors into its x
studying more difficult subjects, such as and y components. Then the components along each axis can be added
together. As shown in Figure 2.8, these sums will be the two perpendicular
inclined-plane problems and equilibrium
components of the resultant, d. The resultant’s magnitude can then be
problems. found by using the Pythagorean theorem, and its direction can be found
by using the inverse tangent function.

FIGURE 2.8

Adding Vectors That Are Not Perpendicular Add the


components of the original displacement vectors to find two components
that form a right triangle with the resultant vector.

d2

d1
d

Differentiated
90 Chapter 3
Instruction
Below Level
Be sure students distinguish between the
Untitled-241 90 5/4/2011 2:51:03 PM

components of d1 and d2 in Figure 2.8. Ask


students to draw each vector and its compo-
nents separately. Then, have them sum the
components in each direction and add the
summed components together to find d.
Repeat with additional examples to help
prepare students for Sample Problem C,
Adding Vectors Algebraically.

90  Chapter 3
PREMIUM CONTENT

Adding Vectors Algebraically Interactive Demo

Classroom Practice
HMDScience.com
Sample Problem C A hiker walks 27.0 km from her base camp
at 35° south of east. The next day, she walks 41.0 km in a direction
65° north of east and discovers a forest ranger’s tower. Find the
Adding Vectors Algebraically  A camper
magnitude and direction of her resultant displacement between walks 4.5 km at 45° north of east then
the base camp and the tower. 4.5 km due south. Find the camper’s total
Select a coordinate system. Then sketch and label displacement.
ANALYZE
each vector.
Ranger’s tower Answer: 3.4 km at 22° south of east
Given: d1 = 27.0 km θ1 = −35°
Tips and Tricks Base A plane flies 118 km at 15.0° south of east
θ1 is negative, because d2 = 41.0 km θ2 = 65° camp
clockwise angles from and then flies 118 km at 35.0° west of
the positive x-axis are Unknown: d= ? θ= ?
conventionally considered north. Find the magnitude and direction
to be negative.
of the total displacement of the plane.
Answer: 81 km at 55° north of east
PLAN Find the x and y components of all vectors.
Make a separate sketch of the displacements for each
day. Use the cosine and sine functions to find the
displacement components.
PROBLEM guide C
= -35°
Use this guide to assign problems.
∆x
cos θ = _ ∆y
sin θ = _ = 27.0 km SE = Student Edition Textbook
d d
(a)
PW = Sample Problem Set I (online)
(a) For day 1: PB = Sample Problem Set II (online)
∆x1 = d1 cos θ1 = (27.0 km) [cos (−35°)] = 22 km
Solving for
∆y1 = d1 sin θ1 = (27.0 km) [sin (−35°)] = −15 km
vector SE Sample, 1–3, 4*;
(b) For day 2: (b)
sum Ch. Rvw. 26, 53*
∆x2 = d2 cos θ2 = (41.0 km) (cos 65°) = 17 km km
PW Sample, 1–5
1.0

∆y2 = d2 sin θ2 = (41.0 km) (sin 65°) = 37 km


=4

Find the x and y components of the total


PB Sample, 1–10
displacement. *Challenging Problem
= 65°
∆xtot = ∆x1 + ∆x2 = 22 km + 17 km = 39 km
∆ytot = ∆y1 + ∆y2 = –15 km + 37 km = 22 km

SOLVE Use the Pythagorean theorem to find the magnitude of the


resultant vector.
d 2 = (∆xtot)2 + (∆ytot)2
d = √(∆x
�������)2 + (∆y )2 =
tot tot √��������
(39 km)2 + (22 km)2 = 45 km
Use a suitable trigonometric function to find the angle.

θ = tan–1 _(
∆ytot
∆xtot ) ( )
22 km = 29° north of east
= tan–1 _
39 km

Continued

Problem Solving Two-Dimensional Motion and Vectors 91

Alternative Approach
ntitled-241 91
In Step 2(a), remind students that they can use 5/4/2011 2:51:04 PM

the following trigonometric formulas as an


alternative approach:
sin (-a) = -sin a
cos (-a) = cos a

Two-Dimensional Motion and Vectors  91


Adding Vectors Algebraically (continued)

 Teach continued
1 A football player runs directly down the field for 35 m before turning to the right at

Answers
an angle of 25° from his original direction and running an additional 15 m before
getting tackled. What is the magnitude and direction of the runner’s total
displacement?
Practice C
1. 49 m at 7.3° to the right of downfield 2. A plane travels 2.5 km at an angle of 35° to the ground and then changes direction
and travels 5.2 km at an angle of 22° to the ground. What is the magnitude and
2. 7.5 km at 26° above the horizontal direction of the plane’s total displacement?
3. 13.0 m at 57° north of east
3. During a rodeo, a clown runs 8.0 m north, turns 55° north of east, and runs
4. 171 km at 34° east of north 3.5 m. Then, after waiting for the bull to come near, the clown turns due east and
runs 5.0 m to exit the arena. What is the clown’s total displacement?

4. An airplane flying parallel to the ground undergoes two consecutive


 Assess and Reteach displacements. The first is 75 km 30.0° west of north, and the second is 155 km
60.0° east of north. What is the total displacement of the airplane?

Assess  Use the Formative Assessment


on this page to evaluate student
mastery of the section.
Reteach  For students who need SECTION 2 FORMATIVE ASSESSMENT
additional instruction, download the
Section Study Guide. Reviewing Main Ideas
1. Identify a convenient coordinate system for analyzing each of the
Response to Intervention  To reassess following situations:
students’ mastery, use the Section Quiz, a. a dog walking along a sidewalk
available to print or to take directly b. an acrobat walking along a high wire
c. a submarine submerging at an angle of 30° to the horizontal
online at HMDScience.com.
2. Find the magnitude and direction of the resultant velocity vector for the
following perpendicular velocities:
a. a fish swimming at 3.0 m/s relative to the water across a river that
moves at 5.0 m/s
b. a surfer traveling at 1.0 m/s relative to the water across a wave that is
traveling at 6.0 m/s
3. Find the vector components along the directions noted in parentheses.
a. a car displaced 45° north of east by 10.0 km (north and east)
b. a duck accelerating away from a hunter at 2.0 m/s2 at an angle of 35° to
the ground (horizontal and vertical)

Critical Thinking
4. Why do nonperpendicular vectors need to be resolved into components
before you can add the vectors together?

Answers
92
to Section Assessment
Chapter 3

1. a. x-axis: forward and backward on 3. a. 7.07 km north, 7.07 km east


Untitled-241 92 sidewalk b. 1.6 m/s2 horizontal, 1.1 m/s2 vertical 5/4/2011 2:51:05 PM

y-axis: left and right on sidewalk 4. because the Pythagorean theorem and the
b. x-axis: forward and backward on rope tangent function can be applied only to
y-axis: up and down right triangles
c. x-axis: horizontal at water level
y-axis: up and down
2. a. 5.8 m/s at 59° downriver from its
intended path
b. 6.1 m/s at 9.5° from the direction the
wave is traveling

92  Chapter 3
SECTION 3 SECTION  3

Projectile Motion Objectives


Recognize examples of
projectile motion.
Plan and Prepare 
Key Term
projectile motion Describe the path of a projectile
as a parabola.
Preview Vocabulary
Visual Vocabulary  Use two tennis balls
Two-Dimensional Motion Resolve vectors into their to illustrate the difference between a
Previously, we showed how quantities such as displacement and velocity components and apply the
kinematic equations to solve
projectile and a falling object. Drop one
were vectors that could be resolved into components. In this section,
these components will be used to understand and predict the motion of problems involving projectile of the balls and point out how it moves
objects thrown into the air. motion. down to the floor along a straight line.
Then throw the other ball toward the
Use of components avoids vector multiplication. wall and point out how this ball moves
How can you know the displacement, velocity, and acceleration of a ball on a curved path until it hits the target.
at any point in time during its flight? All of the kinematic equations could FIGURE 3.1 An object moving in a straight line
be rewritten in terms of vector quantities. However, when an object is
propelled into the air in a direction other than straight up or down, the Motion of a Long Jumper toward the ground is falling. An object
velocity, acceleration, and displacement of the object do not all point in When the long jumper is in the air, his moving on curved path from its point of
the same direction. This makes the vector forms of the equations difficult velocity has both a horizontal and a
vertical component. origin until it reaches a target point is
to solve.
called a projectile.
One way to deal with these situations is to avoid using the complicated
vector forms of the equations altogether. Instead, apply the technique of
resolving vectors into components. Then you can apply the simpler
one-dimensional forms of the equations for each component. Finally, you
can recombine the components to determine the resultant. Teach 
Components simplify projectile motion. TEACH FROM VISUALS
When a long jumper approaches his jump, he runs along a straight line,
which can be called the x-axis. When he jumps, as shown in Figure 3.1, his FIGURE 3.1  Tell students that the long
velocity has both horizontal and vertical components. Movement in this
plane can be depicted by using both the x- and y-axes.
jumper builds up speed in the x direc-
Note that in Figure 3.2(b), a jumper’s velocity vector is resolved into its
tion and jumps, so there is also a
two vector components. This way, the jumper’s motion can be analyzed component of speed in the y direction.
using the kinematic equations applied to one direction at a time.
Ask  Does the angle of takeoff matter
to the jumper? Consider the difference
FIGURE 3.2 between a very small angle (near 0°) and
Components of a Long Jumper’s Velocity
a larger angle (near 45°).
(a) A long jumper’s velocity while sprinting along the Answer: The angle matters because it
runway can be represented by a horizontal vector.
(b) Once the jumper is airborne, the jumper’s
affects how long the jumper stays off
the ground and how far he goes
©Michael Wong/Corbis

velocity at any instant can be described by the v


components of the velocity. horizontally while in the air.
(a) (b)

Differentiated Instruction Two-Dimensional Motion and Vectors 93 Teaching Tip


On the chalkboard, show examples of
Inclusion Next, have students jump forward a short vector components and the kinematic
ntitled-59 93
Visually impaired students may benefit from distance. Again ask, how did you move? up11:47:48 AM
5/13/2011 equations. Show the simplification of
experiencing motions whose vectors have and down, and also forward (horizontally) How the x-direction equations when the x
different components. Go outdoors or to an many components were involved when you component of acceleration is zero.
indoor location where students may move jumped up? one–the vertical direction How
about safely, such as a large, empty lobby or many components were involved when you
gym. Have students jump straight up in place. jumped forward? two–vertical and horizontal
Ask, how did you move? up, then down, or In which case were you a projectile? when I
vertically This demonstrates motion in one jumped forward
dimension.

Two-Dimensional Motion and Vectors  93


FIGURE 3.3
Path without air resistance

 Teach continued Air Resistance Affects Projectile


Motion (a) Without air resistance, the
soccer ball would travel along a parabola.
(b) With air resistance, the soccer ball would Path with air resistance
travel along a shorter path.
Demonstration
(a)
Air Resistance  (b)
Purpose  Show the effects of air
resistance on the flight of a projectile.
Materials  rubber stopper, table-tennis
ball In this section, we will focus on the form of two-dimensional motion
projectile motion the curved path called projectile motion. Objects that are thrown or launched into the air
Procedure  Toss the stopper at an angle that an object follows when thrown, and are subject to gravity are called projectiles. Some examples of projec-
launched, or otherwise projected
of 45°. Have students note the trajec- near the surface of Earth
tiles are softballs, footballs, and arrows when they are projected through
tory. Sketch the parabolic path on the the air. Even a long jumper can be considered a projectile.

chalkboard. Throw the table-tennis ball


at an angle of 45°. Have students note Projectiles follow parabolic trajectories.
The path of a projectile is a curve called a parabola, as shown in
the trajectory. Sketch its path on the Figure 3.3(a). Many people mistakenly think that projectiles eventually fall
chalkboard. Have students compare the straight down in much the same way that a cartoon character does after
two trajectories. Students should note running off a cliff. But if an object has an initial horizontal velocity, there
will be horizontal motion throughout the flight of the projectile. Note that
that the path of the table-tennis ball is
for the purposes of samples and exercises in this book, the horizontal
not symmetrical but has a steeper velocity of projectiles will be considered constant. This velocity would not
descent, demonstrating the effect of be constant if we accounted for air resistance. With air resistance, projec-
tiles slow down as they collide with air particles, as shown in Figure 3.3(b).
air resistance.

Did YOU Know? Projectile motion is free fall with an initial horizontal velocity.
The greatest distance a regulation-size To understand the motion a projectile undergoes, first examine Figure 3.4
baseball has ever been thrown is on the following page. The red ball was dropped at the same instant the
135.9 m, by Glen Gorbous in 1957. yellow ball was launched horizontally. If air resistance is disregarded,
both balls hit the ground at the same time. By examining each ball’s
position in relation to the horizontal lines and to one another, we see that
the two balls fall at the same rate. This may seem impossible because one
is given an initial velocity and the other begins from rest. But if the
motion is analyzed one component at a time, it makes sense.
First, consider the red ball that falls straight down. It has no motion
in the horizontal direction. In the vertical direction, it starts from rest
(vy,i = 0 m/s) and proceeds in free fall. Thus, the kinematic equations from
the chapter “Motion in One Dimension” can be applied to analyze the
vertical motion of the falling ball, as shown on the next page. Note that on
Earth’s surface the acceleration (ay) will equal −g (−9.81 m/s2) because
the only vertical component of acceleration is free-fall acceleration. Note
also that ∆y is negative.

Differentiated
94 Chapter 3
Instruction
inclusion Using your fingers or a stick, manipulate the
Review the general equation for a parabola,
Untitled-59 94 water so that it arcs downward. This is only 5/13/2011 11:47:49 AM

y = ax2 + bx + c, and illustrate a parabola on part of the parabola but students may be able
the board for clarity. Visual learners will to get the general idea of what occurs when a
benefit from time-lapse photography of a projectile passes through part of a parabola.
projectile in motion. Or consider using the
classroom sink and a strobe light to illustrate
the movement of water drops. You could
begin by starting a slow drip from the faucet,
darkening the room, and setting the strobe
light up so that it shines through the drops.

94  Chapter 3
FIGURE 3.4
Vertical Motion of a Projectile That Falls from Rest
vy,f = ay ∆t Vertical Motion of a
Projectile This is a strobe
Teaching Tip
vy,f2 = 2ay ∆y photograph of two table-tennis balls Explain that the vertical motion of a
∆y = _ 1 a (∆t)2 released at the same time. Even
2 y projectile has negative acceleration. This
though the yellow ball is given an
initial horizontal velocity and the red is due to gravity. So, the vertical
Now consider the components of motion of the yellow ball that is ball is simply dropped, both balls fall components of all motions of
launched in Figure 3.4. This ball undergoes the same horizontal displace- at the same rate.
projectiles are affected by gravity and
ment during each time interval. This means that the ball’s horizontal
velocity remains constant (if air resistance is assumed to be negligible). have negative acceleration.
Thus, when the kinematic equations are used to analyze the horizontal
motion of a projectile, the initial horizontal velocity is equal to the
horizontal velocity throughout the projectile’s flight. A projectile’s hori-
zontal motion is described by the following equation. QuickLab
Horizontal Motion of a Projectile Teacher’s Notes
vx = vx,i = constant Dropping the second ball as the first
∆x = vx ∆t leaves the table is tricky. Students
should try this several times in order to
Next consider the initial motion of the launched yellow ball in get the timing right. Holding the second
Figure 3.4. Despite having an initial horizontal velocity, the launched ball
ball just past the edge of the table and
has no initial velocity in the vertical direction. Just like the red ball that
falls straight down, the launched yellow ball is in free fall. The vertical near the path of the first ball works well.
motion of the launched yellow ball is described by the same free-fall You should point out the limitations of
equations. In any time interval, the launched ball undergoes the same
this QuickLab because of human
vertical displacement as the ball that falls straight down. For this reason,
both balls reach the ground at the same time. reaction time.
To find the velocity of a projectile at any point during its flight, find the
vector that has the known components. Specifically, use the Pythagorean
theorem to find the magnitude of the velocity, and use the tangent
function to find the direction of the velocity.

PROJECTILE MOTION
(bl) ©Richard Megna/Fundamental Photographs, New York;

Roll a ball off a table. At the whether the two balls strike the
instant the rolling ball leaves the ground at the same time? Next roll MATERIALS
table, drop a second ball from the one of the balls down a slope. • 2 identical balls
same height above the floor. Do Drop the other ball from the base • slope or ramp
the two balls hit the floor at the of the slope at the instant the first
SAFETY
same time? ball leaves the slope. Which of the Perform this experiment
Try varying the speed at which balls hits the ground first in this away from walls and
you roll the first ball off the table. situation? furniture that can be
damaged.
Does varying the speed affect

Two-Dimensional Motion and Vectors 95

Pre-AP
ntitled-59 95
Explain that the horizontal motion of a 5/13/2011 11:47:50 AM

projectile is independent of its vertical


motion. Have students manipulate the
equations given in the text to show that both
equations are dependent on ∆t. In other
words, show that the the vertical component
has no effect on the horizontal motion and
that the horizontal component has no effect
on the vertical motion.

Two-Dimensional Motion and Vectors  95


PREMIUM CONTENT

Projectiles Launched Horizontally Interactive Demo


HMDScience.com

 Teach continued Sample Problem D The Royal Gorge Bridge in Colorado rises
321 m above the Arkansas River. Suppose you kick a rock
horizontally off the bridge. The magnitude of the rock’s horizontal
Classroom Practice displacement is 45.0 m. Find the speed at which the rock was kicked.

Projectiles Launched ANALYZE Given: ∆y = –321 m ∆x = 45.0 m ay = –g = –9.81 m/s2


Horizontally  People in movies often Unknown: vi = vx = ?
jump from buildings into pools. If a Diagram: The initial velocity
person jumps horizontally from the 10th vector of the rock has
only a horizontal component.
floor (30.0 m) to a pool that is 5.0 m –321 m
Choose the coordinate system
away from the building, with what initial oriented so that the positive
speed must the person jump? y direction points upward and
the positive x direction points
Answer: 2.0 m/s to the right. 45.0 m

PLAN Choose an equation or situation:


PROBLEM guide D Because air resistance can be neglected, the rock’s horizontal velocity
remains constant.
Use this guide to assign problems.
∆x = vx ∆t
SE = Student Edition Textbook Because there is no initial vertical velocity, the following equation
PW = Sample Problem Set I (online) applies.
1 a (∆t)2
∆y = _
PB = Sample Problem Set II (online) 2 y
Solving for: Rearrange the equations to isolate the unknowns:
Note that the time interval is the same for the vertical and horizontal
vx SE Sample, 1–3; Tips and Tricks displacements, so the second equation can be rearranged to solve
Ch. Rvw. 31–32, 51a The value for vx can for ∆t.

be either positive or
PW 6, 7*, 8* 2∆y
∆t = �_
negative because ��
of the square root. a y
PB 9 Because the object is Next rearrange the first equation for vx, and substitute
moving in what has
∆x SE 4; Ch. Rvw. 33 been selected as the the above value of ∆t into the new equation.

(√ )
positive direction, you
PW Sample, 1–2 ∆x = ay
��
choose the positive
answer.
vx = _ _ ∆x
∆t 2∆y
PB 8, 10
∆y SE Ch. Rvw. 51b SOLVE Substitute the values into the equation and solve:


PW 3–4, 5* �����
–9.18 m/s2
vx = __ (45.0 m) = 5.56 m/s
(2) (–321 m)
PB Sample, 1–7
*Challenging Problem
CHECK YOUR To check your work, estimate the value of the time interval for ∆x and
ANSWER solve for ∆y. If vx is about 5.5 m/s and ∆x = 45 m, ∆t ≈ 8 s. If you use
an approximate value of 10 m/s2 for g, ∆y ≈ –320 m, almost identical
to the given value.

Continued

Problem Solving
96 Chapter 3

Deconstructing Problems
_2∆y
Describe the simplification of the formula
Untitled-59 96
​  a  ​ = (∆t)2 Simplify. 5/13/2011 11:47:51 AM

y
as follows:

√ 

2∆y
∆y = ​ _21  ​ay (∆t)2   Given ​  _
​  a  ​  ​ = ∆t Take square root
y
of each side.
_ ∆y _ ​ _21  ​ ay (∆t )2
​ _1   ​ = ​  _1  ​  Divide both sides by ​ _21  ​ ay.

​ 2  ​ ay ​ 2  ​ ay

96  Chapter 3
Projectiles Launched Horizontally (continued)

Answers
1. A baseball rolls off a 0.70 m high desk and strikes the floor 0.25 m away from the Practice D
base of the desk. How fast was the ball rolling?
1. 0.66 m/s
2. A cat chases a mouse across a 1.0 m high table. The mouse steps out of the way,
2. 4.9 m/s
and the cat slides off the table and strikes the floor 2.2 m from the edge of the table.
When the cat slid off the table, what was its speed? 3. 7.6 m/s
3. A pelican flying along a horizontal path drops a fish from a height of 4 m. The fish 4. 5.6 m
travels 8.0 m horizontally before it hits the water below. What is the pelican’s
speed?

4. If the pelican in item 3 was traveling at the same speed but was only 2.7 m above TEACH FROM VISUALS
the water, how far would the fish travel horizontally before hitting the water
below? FIGURE 3.5  Point out to students that
from the time immediately after firing
until it hits the ground, the projectile
Use components to analyze objects launched at an angle. FIGURE 3.5 follows a parabolic path. Emphasize that
Let us examine a case in which a projectile is launched at an angle to the the vector vi does not represent any
Components of Initial Velocity
horizontal, as shown in Figure 3.5. The projectile has an initial vertical
component of velocity as well as a horizontal component of velocity.
An object is projected with an initial part of the path of the projectile but
velocity, vi, at an angle of θ. Resolve the
Suppose the initial velocity vector makes an angle θ with the horizontal. initial velocity into its x and y components.
only the direction and magnitude of its
Again, to analyze the motion of such a projectile, you must resolve the Then, the kinematic equations can be initial velocity.
initial velocity vector into its components. The sine and cosine functions applied to describe the motion of the
can be used to find the horizontal and vertical components of the initial projectile throughout its flight.
Ask  What is the acceleration of a
velocity. projectile just before it hits the ground?
vx,i = vi cos θ and vy,i = vi sin θ
vi
Answer: −9.81 m/s2, the same as at any
We can substitute these values for vx,i and vy,i into the kinematic other time during the flight
equations to obtain a set of equations that can be used to analyze the
motion of a projectile launched at an angle.

Projectiles Launched at an Angle


vx = vx,i = vi cos θ = constant
∆x = (vi cos θ)∆t
vy,f = vi sin θ + ay ∆t
vy,f2 = vi2 (sin θ)2 + 2ay ∆y
∆y = (vi sin θ)∆t + _ 1 a (∆t)2
2 y

As we have seen, the velocity of a projectile launched at an angle to


the ground has both horizontal and vertical components. The vertical
motion is similar to that of an object that is thrown straight up with an
initial velocity.

Two-Dimensional Motion and Vectors 97

Reality Check
ntitled-59 97
Students often memorize equations without 5/13/2011 11:47:52 AM

taking the time to understand them.


Encourage students to visualize how each of
these equations is a form of one of the
kinematic equations they learned earlier.
Explain that solving projectile problems is
done by applying the kinematic equations
separately in each direction.

Two-Dimensional Motion and Vectors  97


Projectiles Launched at an Angle

 Teach continued Sample Problem E A zookeeper finds an escaped monkey on


a pole. While aiming her tranquilizer gun at the monkey, she
kneels 10.0 m from the pole, which is 5.00 m high. The tip of her gun is 1.00 m
Classroom Practice above the ground. At the moment the zookeeper shoots, the monkey drops a banana.
The dart travels at 50.0 m/s. Will the dart hit the monkey, the banana, or neither one?
Projectiles Launched at an Angle 
ANALYZE Select a coordinate system.
A golfer practices driving balls off a cliff The positive y-axis points up, 10.0 m
and into the water below. The edge of and the positive x-axis points
the cliff is 15 m above the water. If the along the ground toward the
4.00 m
pole. Because the dart leaves
golf ball is launched at 51 m/s at an angle the gun at a height of 1.00 m,
of 15°, how far does the ball travel the vertical distance is 4.00 m. 1.00 m

horizontally before hitting the water?


PLAN Use the inverse tangent function to find the angle of the dart with the
(See Appendix A for hints on solving
x-axis.
quadratic equations.)
Answer: 1.7 × 10 m
2
∆y
∆x( ) (
4.00 m = 21.8°
θ = tan–1 _ = tan–1 _
10.0 m )
Choose a kinematic equation to solve for time.
Rearrange the equation for motion along the x-axis to isolate ∆t, the
PROBLEM guide e unknown, the time the dart takes to travel the horizontal distance.
∆x = (vi cos θ)∆t
Use this guide to assign problems.
∆t = _ ∆x = __ 10.0 m
SE = Student Edition Textbook = 0.215 s
vi cos θ (50.0 m/s)(cos 21.8°)
PW = Sample Problem Set I (online)
PB = Sample Problem Set II (online) SOLVE Find out how far each object will fall during this time.
TSI Graphics
Use the free-fall kinematic equation. For the banana, vi = 0. Thus:
HRW • Holt Physics
Solving for: 1 a (∆t)2 = _ 1 (–9.81 m/s2)(0.215
∆y = _
b y
PH99PE-C03-003-013-A
s)2 = –0.227 m
∆y/∆x SE Sample, 1–3; 2 2
The dart has an initial vertical component of velocity of vi sin θ, so:
Ch. Rvw. 34a, 1 a (∆t)2
35–36, 55a, 56b, ∆yd = (vi sin θ)∆t + _
2 y
59–60, 62a, 62c 1 (–9.81 m/s2)(0.215 s)2
∆yd = (50.0 m/s)(sin 21.8°)(0.215 s) + _
PW 4b, 5, 7–8 2
∆yd = 3.99 m – 0.227 m = 3.76 m
PB 6
Find the final height of both the banana and the dart.
νi SE 4; Ch. Rvw.
47a*, 48a, 49*, ybanana, f = yb,i + ∆yb = 5.00 m + (−0.227 m) =
56a, 61 4.77 m above the ground
PW Sample, 1–3, 4a
ydart, f = yd,i + ∆yd = 1.00 m + 3.76 m =
PB 8, 10
∆t SE Ch. Rvw. 34b, 4.76 m above the ground
47*b, 55b, 62b The dart hits the banana. The slight difference is due to rounding.
PB 7, 9
νf SE Ch. Rvw. 47c*, Continued
48b Problem
98
Solving
Chapter 3

θ PW 6
Alternative Approaches
PB Sample, 1–5
Point out that there are different ways to
Untitled-59 98 5/13/2011 11:47:53 AM

*Challenging Problem
solve comparison questions such as item 1 in
Practice E. In this case, one option is to solve
for ∆y and then compare the calculated value
of ∆y with the vertical distance given (2.5 m).
Another option is to solve for ∆x and
compare the calculated value of ∆x with the
horizontal distance given (4.0 m).

98  Chapter 3
Projectiles Launched at an Angle (continued)

Answers
1. In a scene in an action movie, a stuntman jumps from the top of one building to Practice E
the top of another building 4.0 m away. After a running start, he leaps at a velocity 1. yes, ∆y = −2.3 m
of 5.0 m/s at an angle of 15° with respect to the flat roof. Will he make it to the other
roof, which is 2.5 m lower than the building he jumps from? 2. 35.1 m
2. A golfer hits a golf ball at an angle of 25.0° to the ground. If the golf ball covers a
3. 2.0 s; 4.8 m
horizontal distance of 301.5 m, what is the ball’s maximum height? (Hint: At the 4. 6.2 m/s
top of its flight, the ball’s vertical velocity component will be zero.)

3. A baseball is thrown at an angle of 25° relative to the ground at a speed of 23.0 m/s.
If the ball was caught 42.0 m from the thrower, how long was it in the air? How high
above the thrower did the ball travel?
Assess and Reteach 
4. Salmon often jump waterfalls to reach their breeding grounds. One salmon starts Assess  Use the Formative Assessment
2.00 m from a waterfall that is 0.55 m tall and jumps at an angle of 32.0°. What must on this page to evaluate student
be the salmon’s minimum speed to reach the waterfall?
mastery of the section.
Reteach  For students who need
additional instruction, download the
SECTION 3 FORMATIVE ASSESSMENT Section Study Guide.
Reviewing Main Ideas Response to Intervention  To reassess
1. Which of the following exhibit parabolic motion? students’ mastery, use the Section Quiz,
a. a flat rock skipping across the surface of a lake available to print or to take directly
b. a three-point shot in basketball
online at HMDScience.com.
c. a space shuttle while orbiting Earth
d. a ball bouncing across a room
e. a life preserver dropped from a stationary helicopter
2. During a thunderstorm, a tornado lifts a car to a height of 125 m above the ground. Increasing in
strength, the tornado flings the car horizontally with a speed of 90.0 m/s. How long does the car
take to reach the ground? How far horizontally does the car travel before hitting the ground?

Interpreting Graphics
3. An Alaskan rescue plane drops a package FIGURE 3.6
of emergency rations to a stranded party of
explorers, as illustrated in Figure 3.6. The plane Dropping a Package
is traveling horizontally at 30.0 m/s at a height
of 200.0 m above the ground. plane = 30.0 m/s
a. What horizontal distance does the package
fall before landing?
b. Find the velocity of the package just before
it hits the ground. 200.0 m

Answers to Section Assessment Two-Dimensional Motion and Vectors 99


PHYSICS
1. a, b, d Spec. Number PH 99 PE C03-003-010-A
Boston Graphics, Inc.
ntitled-59 99 2. 5.05 s; 454 m 617.523.1333
5/13/2011 11:47:54 AM

3. a. 192 m
b. 69.4 m/s at 64.4° below the horizontal

Two-Dimensional Motion and Vectors  99


SECTION  4 SECTION 4

 Plan and Prepare


Objectives
Describe situations in terms of
frame of reference.
Relative Motion
Preview Vocabulary Solve problems involving
relative velocity.
Frames of Reference
Visual Vocabulary  Ask students to If you are moving at 80 km/h north and a car passes you going 90 km/h
north, to you the faster car seems to be moving north at 10 km/h.
imagine a person walking inside a moving
Someone standing on the side of the road would measure the velocity of
train. Then ask students to imagine the faster car as 90 km/h north. This simple example demonstrates that
another person outside the train who velocity measurements depend on the frame of reference of the observer.
is moving faster than the person inside.
Let students brainstorm descriptions Velocity measurements differ in different frames of reference.
of how the two people perceive each Observers using different frames of reference may measure different
displacements or velocities for an object in motion. That is, two observers
other’s motions . Then explain that in moving with respect to each other would generally not agree on some
such scenarios where a combination features of the motion.
of velocities is involved, we use a term Consider a stunt dummy that is dropped from an airplane flying
called relative velocity, in which the horizontally over Earth with a constant velocity. As shown in Figure 4.1(a),
a passenger on the airplane would describe the motion of the dummy as
velocity of one object is compared a straight line toward Earth. An observer on the ground would view the
relative to the velocity of another trajectory of the dummy as that of a projectile, as shown in Figure 4.1(b).
object. Relative to the ground, the dummy would have a vertical component of
velocity (resulting from free-fall acceleration and equal to the velocity
FIGURE 4.1
measured by the observer in the airplane) and a horizontal component of
Frames of Reference When velocity given to it by the airplane’s motion. If the airplane continued to
 Teach viewed from the plane (a), the stunt
dummy (represented by the maroon dot)
move horizontally with the same velocity, the dummy would enter the
swimming pool directly beneath the airplane (assuming negligible air
falls straight down. When viewed from a resistance).
stationary position on the ground (b),
TEACH FROM VISUALS the stunt dummy follows a parabolic
projectile path.
FIGURE 4.1  Tell students that this
diagram shows what would happen (a)
if there were no air resistance.
Ask  How would the two diagrams
change if air resistance were included?
Answer: In the first diagram, the object (b)
would appear to fall down and backward
as viewed by the pilot. In the second
diagram, the object would follow a
shortened path as viewed by the
observer on the ground. (At any instant
after release, both the x-component
and the y-component would be smaller,
but the suppression of horizontal Differentiated
100 Chapter 3
Instruction
motion has the dominant effect on the
PHYSICS
trajectory.) Inclusion Examples: Spec. Number PH 99 PE C03-004-001-A
Boston Graphics, Inc.
Matching one moving object with another
Untitled-307 100 1. moving boat 617.523.1333 5/6/2011 11:58:21 AM

moving object can help visual learners 2. moving elevator


understand relative velocity. Put a list of 3. motion of a planet in the Solar System
moving objects such as the ones to the right
4. walking passenger in a train
on the board. Ask students to name some
moving objects that can be paired with one 5. rolling of a ball bearing
of the examples in order to establish a case for
relative velocity. For example, students can
match the motion of a moving boat with
a person swimming in the water.

100  Chapter 3
Relative Velocity
The case of the faster car overtaking your car was easy to solve with a
minimum of thought and effort, but you will encounter many situations in
Teaching Tip
which a more systematic method of solving such problems is beneficial. To Relative velocity can also be shown
develop this method, write down all the information that is given and that as the difference of two vectors.
you want to know in the form of velocities with subscripts appended.
vfs = vfe – vse
vse = +80 km/h north (Here the subscript se means the velocity
of the slower car with respect to Earth.)
Another way of stating this
equation is that the relative velocity
vfe = +90 km/h north (The subscript fe means the velocity
of the fast car with respect to Earth.) of one moving object to another is the
difference between their velocities
We want to know vfs, which is the velocity of the fast car with respect
relative to some common reference
to the slower car. To find this, we write an equation for vfs in terms of the
other velocities, so on the right side of the equation the subscripts start point.
with f and eventually end with s. Also, each velocity subscript starts with You may want to demonstrate on
the letter that ended the preceding velocity subscript.
the board that this equation works for
vfs = vfe + ves noncollinear velocities, as in Sample
The boldface notation indicates that velocity is a vector quantity. This
Problem F on the next page.
approach to adding and monitoring subscripts is similar to vector addi-
tion, in which vector arrows are placed head to tail to find a resultant.
We know that ves = −vse because an observer in the slow car per-
ceives Earth as moving south at a velocity of 80 km/h while a stationary Answers
observer on the ground (Earth) views the car as moving north at a veloc- Conceptual Challenge
ity of 80 km/h. Thus, this problem can be solved as follows:
1. Greater than, because the elevator is
vfs = vfe + ves = vfe − vse accelerating upward toward the ball
vfs = (+90 km/h north) − (+80 km/h north) = +10 km/h north as it falls
When solving relative velocity problems, follow the above technique
2. The plane’s velocity is slower relative
for writing subscripts. The particular subscripts will vary depending on to the moving carrier when it
the problem, but the method for ordering the subscripts does not approaches from the stern.
change. A general form of the relative velocity equation is vac = vab + vbc.
This general form may help you remember the technique for writing
subscripts.

Conceptual Challenge
1. Elevator Acceleration A boy 2. Aircraft Carrier Is the velocity of a
bounces a rubber ball in an elevator that plane relative to an aircraft carrier slower
©Ross Harrison Koty/Getty Images

is going down. If the boy drops the ball when it approaches from the stern (rear)
as the elevator is slowing down, is the or from the bow (front)?
magnitude of the ball’s acceleration
relative to the elevator less than or
greater than the magnitude of its
acceleration relative to the ground?

Two-Dimensional Motion and Vectors 101

Untitled-307 101 5/6/2011 11:58:24 AM

Two-Dimensional Motion and Vectors  101


Relative Velocity

 Teach continued Sample Problem F A boat heading north crosses a wide river
with a velocity of 10.00 km/h relative to the water. The river has a
uniform velocity of 5.00 km/h due east. Determine the boat’s
Classroom Practice velocity with respect to an observer on shore.

Relative Velocity  A plane flies ANALYZE Given: vbw = 10.00 km/h due north vwe

northeast at an airspeed of 563.0 km/h. (velocity of the boat, b, with


respect to the water, w)
(Airspeed is the speed of an aircraft
vwe = 5.00 km/h due east N
relative to the air.) A 48.0 km/h wind is vbw v be
(velocity of the water, w, with W E
blowing to the southeast. What is the respect to Earth, e)
S
plane’s velocity relative to the ground?
Unknown: vbe = ? θ=?
Answer: 565.0 km/h at 40.1° north Diagram: See the diagram on the right.
of east
PLAN Choose an equation or situation:
The wind shifts to blow 63.0 km/h
To find vbe , write the equation so that the subscripts on the right start
toward the southwest. What is the
with b and end with e.
plane’s velocity relative to the ground if
vbe = vbw + vwe
the plane’s airspeed remains constant?
Answer: 500.0 km/h northeast We use the Pythagorean theorem to calculate the magnitude of the
resultant velocity and the tangent function to find the direction.DTSI Graphics
HRW • Holt Physics
(vbe)2 = (vbw)2 + (vwe)2 PH99PE-C03-004-002b-A
vwe
tan θ = _
v bw
Rearrange the equations to isolate the unknowns:

vbe = √������
(vbw)2 + (vwe)2
vwe
θ = tan−1 _
vbw( )
SOLVE Substitute the known values into the equations and solve:

vbe = √�����������
(10.00 km/h)2 + (5.00 km/h)2

vbe = 11.18 km/h

(
5.00 km/h
θ = tan−1 _
10.0 km/h )
θ = 26.6°

CHECK The boat travels at a speed of 11.18 km/h in the direction 26.6° east of
YOUR WORK north with respect to Earth.

Continued

Problem Solving
102 Chapter 3

Take It Further
Tell students that the rules for determining
Untitled-307 102 5/6/2011 11:58:25 AM

relative velocity break down as one ap-


proaches speeds close to the speed of light.
The speed of light, 3.00 × 108 m/s, is constant
in a vacuum. If an object, such as a spaceship,
approached Earth at 0.5 the speed of light, the
lights from the headlights of the spaceship
would not approach Earth at 1.5 times the
speed of light. Instead, they would travel at
the speed of light.

102  Chapter 3
Relative Velocity (continued)

PROBLEM guide F
1. A passenger at the rear of a train traveling at 15 m/s relative to Earth throws a Use this guide to assign problems.
baseball with a speed of 15 m/s in the direction opposite the motion of the train.
What is the velocity of the baseball relative to Earth as it leaves the thrower’s hand? SE = Student Edition Textbook
PW = Sample Problem Set I (online)
2. A spy runs from the front to the back of an aircraft carrier at a velocity of 3.5 m/s. If
the aircraft carrier is moving forward at 18.0 m/s, how fast does the spy appear to PB = Sample Problem Set II (online)
be running when viewed by an observer on a nearby stationary submarine?
Solving for:
3. A ferry is crossing a river. If the ferry is headed due north with a speed of 2.5 m/s v, θ SE Sample, 1–4;
relative to the water and the river’s velocity is 3.0 m/s to the east, what will the Ch. Rvw. 43a,
boat’s velocity relative to Earth be? (Hint: Remember to include the direction in
describing the velocity.)
44a–b, 46a–b,
57a–b
4. A pet-store supply truck moves at 25.0 m/s north along a highway. Inside, a dog PW Sample 1
moves at 1.75 m/s at an angle of 35.0° east of north. What is the velocity of the dog
relative to the road? PB 7, 10
∆t SE Ch. Rvw. 45,
50a–b, 52, 54, 58
PW 2, 4, 6
PB Sample, 1–5
∆y/∆x SE Ch. Rvw. 43b
SECTION 4 FORMATIVE ASSESSMENT PW 5
Reviewing Main Ideas PB 6, 8–9
1. A woman on a 10-speed bicycle travels at 9 m/s relative to the ground as *Challenging Problem
she passes a little boy on a tricycle going in the opposite direction. If the
boy is traveling at 1 m/s relative to the ground, how fast does the boy
appear to be moving relative to the woman?
2. A girl at an airport rolls a ball north on a moving walkway that moves east.
If the ball’s speed with respect to the walkway is 0.15 m/s and the walk- Answers
way moves at a speed of 1.50 m/s, what is the velocity of the ball relative
to the ground? Practice F
1. 0 m/s
Critical Thinking
2. 14.5 m/s (in the direction that the
3. Describe the motion of the following objects if they are observed from the
aircraft carrier is moving)
stated frames of reference:
a. a person standing on a platform viewed from a train traveling north 3. 3.90 m/s at (4.0 × 101 )° north of east
b. a train traveling north viewed by a person standing on a platform 4. 26.4 m/s at 2.17° east of north
c. a ball dropped by a boy walking at a speed of 1 m/s viewed by the boy
d. a ball dropped by a boy walking 1 m/s as seen by a nearby viewer who
is stationary
Assess and Reteach 
Assess  Use the Formative Assessment
on this page to evaluate student
Answers to Section Assessment Two-Dimensional Motion and Vectors 103
mastery of the section.
Reteach  For students who need
1. 10 m/s (in the opposite direction)
additional instruction, download the
ntitled-307 103 2. 1.51 m/s at 5.7° north of east 5/6/2011 11:58:26 AM
Section Study Guide.
3. a. south with a speed equal to the
Response to Intervention  To reassess
train’s speed
students’ mastery, use the Section Quiz,
b. moves north available to print or to take directly
c. appears to fall straight down online at HMDScience.com.
d. moves in a parabola

Two-Dimensional Motion and Vectors  103


PHYSICS ON THE EDGE
PHYSICS ON THE EDGE
Teaching Tip
This feature builds on the feature
“Special Relativity and Time Dilation.” In
Special Relativity
that feature, the assumption that the
speed of light is the same for all and Velocities
observers was used to explain why time
In the chapter “Two-Dimensional Motion and Vectors,” you learned that
measurements depend on an observer’s velocity measurements are not absolute; every velocity measurement
frame of reference. In this feature, this depends on the frame of reference of the observer with respect to the
assumption and its consequences are moving object. For example, imagine that someone riding a bike toward
you at 25 m/s (v) throws a softball toward you. If the bicyclist measures the
discussed in greater detail. softball’s speed (u' ) to be 15 m/s, you would perceive the ball to be moving
The feature begins by comparing the toward you at 40 m/s (u) because you have a different frame of reference
behavior of light with a typical case to than the bicyclist does. This is expressed mathematically by the equation
u = v + u', which is also known as the classical addition of velocities.
show how the two differ. Next, the
need to revise the classical addition of FIGURE 1 The Speed of Light
velocities is discussed. Finally, Einstein’s
Nothing Can Travel Faster As stated in the feature “Special Relativity and Time Dilation,” according to
relativistic addition of velocities is Than the Speed of Light Einstein’s special theory of relativity, the speed of light is absolute, or
introduced, along with a discussion of According to Einstein’s relativistic independent of all frames of reference. If, instead of a softball, the bicyclist
equation for the addition of velocities, were to shine a beam of light toward you, both you and the bicyclist would
how all cases are covered with this material particles can never reach the measure the light’s speed as 3.0 × 108 m/s. This would remain true even if
equation. speed of light. the bicyclist were moving toward you at 99 percent of the speed of light.
Today, there is overwhelming Thus, Einstein’s theory requires a different approach to the addition of
velocities. Einstein’s modification of the classical formula, which he
physical evidence that the speed of derived in his 1905 paper on special relativity, covers both the case of the
light is absolute. Experiments in particle softball and the case of the light beam.
accelerators, in which particles reach
v + u'
speeds very close to c, support the u = __
1 + (vu'/c2)
relativistic rather than the classical
addition of velocities. In the equation, u is the velocity of an object in a reference frame, u' is
the velocity of the same object in another reference frame, v is the veloc-
ity of one reference frame relative to another, and c is the speed of light.

Teaching Tip The Universality of Einstein’s Equation


The constancy of the speed of light is How does Einstein’s equation cover both cases? First we shall consider the
bicyclist throwing a softball. Because c2 is such a large number, the vu'/c2
difficult for some students to grasp,
term in the denominator is very small for velocities typical of our everyday
because we have no direct experience experience. As a result, the denominator of the equation is essentially
of this phenomenon. Use examples to equal to 1. Hence, for speeds that are small compared with c, the two
theories give nearly the same result, u = v + u' , and the classical addition
familiarize your students with Einstein’s
of velocities can be used.
theory of the constancy of the speed of
However, when speeds approach the speed of light, vu'/c2 increases,
light. For example, ask students to and the denominator becomes greater than 1 but never more than 2.
compare the speed of sound and light
waves as viewed by two different 104 Chapter 3

observers, one at rest and one moving


toward the source of the waves.
Untitled-305 104 5/6/2011 11:57:15 AM

104  Chapter 3
When this occurs, the difference between the two theories becomes
Extension
significant. For example, if a bicyclist moving toward you at 80 percent of • Have students research particle-
the speed of light were to throw a ball to you at 70 percent of the speed of
light, you would observe the ball moving toward you at about 96 percent
accelerator experiments involving
of the speed of light rather than the 150 percent of the speed of light particles traveling at speeds close to c.
predicted by classical theory. In this case, the difference between the Their reports should include a discus-
velocities predicted by each theory cannot be ignored, and the relativistic
addition of velocities must be used.
sion of how observations support the
In this last example, it is significant that classical addition predicts a
relativistic addition of velocities.
speed greater than the speed of light (1.5c), while the relativistic addition • The ether was originally conceived as
predicts a speed less than the speed of light (0.96c). In fact, no matter how the medium through which light waves
close the speeds involved are to the speed of light, the relativistic equa-
tion yields a result less than the speed of light, as seen in Figure 2. traveled. Have students investigate the
How does Einstein’s equation cover the second case, in which the concept of the ether and the
bicyclist shines a beam of light toward you? Einstein’s equation predicts Michelson-Morley experiment, which
that any object traveling at the speed of light (u' = c) will appear to travel was intended to detect the ether.
at the speed of light (u = c) for an observer in any reference frame:
Then have a class discussion about why
u = __
v + u' v+c v+c
= __ = _ = _ = c
v+c the concept of the ether was originally
1 + (vu'/c2) 1 + (vc/c2) 1 + (v/c) (c + v)/c believed to be necessary and how the
This corresponds with our earlier statement that the bicyclist measures Michelson-Morley experiment and the
the beam of light traveling at the same speed that you do, 3.0 × 108 m/s, special theory of relativity affected
even though you have a different reference frame than the bicyclist does. the theory of the ether.
This occurs regardless of how fast the bicycle is moving because v (the
bicycle’s speed) cancels from the equation. Thus, Einstein’s relativistic
equation successfully covers both cases. So, Einstein’s equation is a more
general case of the classical equation, which is simply the limiting case.

FIGURE 2

CLASSICAL AND RELATIVISTIC ADDITION OF VELOCITIES


Classical Relativistic
c = 299 792 458 m/s
addition addition
Speed between Speed measured Speed measured Speed measured
frames (v) in A (u' ) in B (u) in B (u)
©Courtesy of the Archives, California Institute of Technology

25 m/s 15 m/s 40 m/s 40 m/s

100 000 m/s 100 000 m/s 200 000 m/s 200 000 m/s

50% of c 50% of c 299 792 458 m/s 239 833 966 m/s

90% of c 90% of c 539 626 424 m/s 298 136 146 m/s

99.99% of c 99.99% of c 599 524 958 m/s 299 792 457 m/s

Two-Dimensional Motion and Vectors 105

Untitled-305 105 5/6/2011 11:57:17 AM

Two-Dimensional Motion and Vectors  105


Careers in Physics
CAREERS IN PHYSICS
Kinesiologist
Lisa Griffin was born in Toronto, Canada,
and grew up in a small town in Ontario.
Kinesiologist
Griffin says that she has always been
interested in “the nature of reality and

H
the relationship between the mind and ow does the body move? This question is just one
the body.” As a high school student of the many that kinesiology continually asks. To
learn more about kinesiology as a career, read the
visiting her public library, she first interview with Lisa Griffin, who teaches in the Department
learned that the link between the mind of Kinesiology and Health Education at the University of
and the body was the central nervous Texas at Austin. Lisa Griffin applies an electrical stimulus to a
system (CNS). As an undergraduate at nerve in a patient’s wrist. This experiment
What training did you receive in order to tested the best patterns of stimulation to
the University of Guelph, she was become a kinesiologist? recreate movement in paralyzed hands.
inspired by Dr. John Brooke, who taught I received a B.Sc. degree in human kinetics with a minor in
that the spinal cord is also part of the biochemistry and M.Sc. and Ph.D. degrees in neuroscience.
Kinesiology typically covers motor control, biomechanics,
brain and has its own forms of “thinking” and exercise physiology. People who work in these branches and a computer data acquisition and analysis program.
and memory. are known as neuroscientists, biomechanists, and For example, the muscles of the thumb produce force in
physiologists, respectively. both x and y directions. We record the x and y forces on two
different channels, and then we calculate the resultant force
What makes kinesiology interesting online so that we can view the net output during contraction.
to you?
The field of kinesiology allows me to explore how the central What are your most and least favorite
nervous system (CNS) controls human movement. Thus we things about your work?
work with people, and the findings of our work can be used My favorite thing is coming up with new ideas and working
to help others. with students who are excited about their work. The thing I
would most like to change is the amount
What is the nature of your research? of time it takes to get the results of
We record force output and single motor unit firing patterns the experiments after you think of
from the muscles of human participants during fatigue and the ideas.
training. We then use these frequency patterns to stimulate
their hands artificially with electrical stimulation. We are What advice would you
offer to students who
working toward developing an electrical stimulation system
are interested in this
that people with paralysis could use to generate limb field?
movement. This could help many who have spinal cord
Do not underestimate the
injuries from accidents or brain damage from stroke.
depth of the questions
that can be addressed
How does your work address two-
dimensional motion and vectors? with human participants.
I investigate motor unit firing frequencies required to
generate force output from muscle over time. Thus we
record muscle contraction with strain gauge force Lisa Griffin
transducers, bridge amplifiers, an analog to digital converter,

106

Untitled-787 106 6/3/2011 4:22:43 PM

106  Chapter 3
C h a p t e r s u m m a ry
CHAPTER 3 Summary
Teaching Tip
SECTION 1 Introduction to Vectors KEY TERMS
Ask students to prepare a concept map
• A scalar is a quantity completely specified by only a number with appropriate scalar for the chapter. The concept map
units, whereas a vector is a quantity that has magnitude and direction. vector should include most of the vocabulary
• Vectors can be added graphically using the triangle method of addition, in resultant
which the tail of one vector is placed at the head of the other. The resultant terms, along with other integral terms
is the vector drawn from the tail of the first vector to the head of the last or concepts.
vector.

SECTION 2 Vector Operations KEY TERM

• The Pythagorean theorem and the inverse tangent function can be used to components of a vector
find the magnitude and direction of a resultant vector.
• Any vector can be resolved into its component vectors by using the sine
and cosine functions.

SECTION 3 Projectile Motion KEY TERM

• Neglecting air resistance, a projectile has a constant horizontal velocity and projectile motion
a constant downward free-fall acceleration.
• In the absence of air resistance, projectiles follow a parabolic path.

SECTION 4 Relative Motion


• If the frame of reference is denoted with subscripts (vab is the velocity of
object or frame a with respect to object or frame b), then the velocity of an
object with respect to a different frame of reference can be found by adding
the known velocities so that the subscript starts with the letter that ends
the preceding velocity subscript: vac = vab + vbc.
• If the order of the subscripts is reversed, there is a change in sign; for
example, vcd = −vdc.

VARIABLE SYMBOLS DIAGRAM SYMBOLS

Quantities Units displacement vector

d (vector) displacement m meters velocity vector

v (vector) velocity m/s meters /second acceleration vector

a (vector) acceleration m/s2 meters /second2 resultant vector

∆x (scalar) horizontal component m meters component

∆y (scalar) vertical component m meters DTSI Graphics


Problem Solving
HRW • Holt Physics
PH99PE-C03-CHS-003-A
See Appendix D : Equations for a summary
of the equations introduced in this chapter. If
you need more problem-solving practice, see
Appendix I: Additional Problems.

Chapter Summary 107

ntitled-237 107 5/4/2011 2:48:50 PM

Two-Dimensional Motion and Vectors  107


C HAPTER RE V I E W
CHAPTER 3 Review
Answers
1. A scalar represents the magnitude Vectors and the Graphical 8. A dog searching for a bone walks 3.50 m south, then
8.20 m at an angle of 30.0° north of east, and finally
of a physical quantity. Method 15.0 m west. Use graphical techniques to find the
dog’s resultant displacement vector.
2. no, must be equal and opposite REVIEWING MAIN IDEAS
9. A man lost in a maze makes three consecutive
3. Speed is the magnitude of velocity. 1. The magnitude of a vector is a scalar. Explain this displacements so that at the end of the walk he is
4. 30 m/s east statement. back where he started, as shown below. The first
displacement is 8.00 m westward, and the second is
5. no, because the scalar has no 2. If two vectors have unequal magnitudes, can their
13.0 m northward. Use the graphical method to find
sum be zero? Explain.
direction the third displacement.
3. What is the relationship between instantaneous
6. a. 5.00 units at 53.1° below the speed and instantaneous velocity?
positive x-axis
4. What is another way of saying −30 m/s west?
b. 5.00 units at 53.1° above the
5. Is it possible to add a vector quantity to a scalar
positive x-axis quantity? Explain.
c. 8.54 units at 69.4° below the 6. Vector A is 3.00 units in length and points along the
positive x-axis positive x-axis. Vector B is 4.00 units in length and
points along the negative y-axis. Use graphical
d. 5.00 units at 127° clockwise from methods to find the magnitude and direction of the
the positive x-axis following vectors:
CONCEPTUAL QUESTIONS
a. A + B
7. a. 5.20 m at 60.0° above the b. A − B 10. If B is added to A, under what conditions does the
positive x-axis c. A + 2B resultant have the magnitude equal to A + B?
PHYSICS
Spec. Number
d. B − A
b. 3.00 m at 30.0° below the 11. Give an example of a moving object that has a Boston Graphic
7. Each of the displacement vectors A and B shown in 617.523.1333
positive x-axis velocity vector and an acceleration vector in the same
the figure below has a magnitude of 3.00 m. direction and an example of one that has velocity and
c. 3.00 m at 150° counterclockwise Graphically find the following: acceleration vectors in opposite directions.
from the positive x-axis a. A + B
b. A − B 12. A student accurately uses the method for combining
d. 5.20 m at 60.0° below the positive c. B − A vectors. The two vectors she combines have
magnitudes of 55 and 25 units. The answer that she
x-axis d. A − 2B
gets is either 85, 20, or 55. Pick the correct answer,
8. 7.9 m at 4.3° north of west and explain why it is the only one of the three that can
be correct.
9. 15.3 m at 58.4° south of east
13. If a set of vectors laid head to tail forms a closed
10. when the vectors point in the same polygon, the resultant is zero. Is this statement true?
direction Explain your reasoning.
11. a car moving straight and speeding
up, a car moving straight and
slowing down
12. 55; the maximum value of the vector
sum is 80 units, the minimum value is
30 units
13. Yes, the distance from the tail of the
108 Chapter 3
first vector to the head of the last
vector is zero.

Untitled-237 108 5/4/2011 2:48:51 PM

108  Chapter 3
C HAPTER RE V I E W
Vector Operations 25. A person walks 25.0° north of east for 3.10 km. How
far would another person walk due north and due 14. Yes, the second component could
REVIEWING MAIN IDEAS east to arrive at the same location? be nonzero.
14. Can a vector have a component equal to zero and still For problem 26, see Sample Problem C. 15. No, the hypotenuse is always greater
have a nonzero magnitude?
26. A person walks the path shown below. The total trip than the legs.
15. Can a vector have a component greater than its consists of four straight-line paths. At the end of the
walk, what is the person’s resultant displacement
16. Vector addition is combining vectors
magnitude?
measured from the starting point? to find a resultant vector. Vector
16. Explain the difference between vector addition and
vector resolution.
resolution is breaking a vector into
100.0 m
its component vectors.
17. How would you add two vectors that are not perpen-
N
dicular or parallel?
? 17. by resolving both vectors into their
300.0 m
W E vector components, adding the
CONCEPTUAL QUESTIONS corresponding components together,
200.0 m S
18. If A + B equals 0, what can you say about the compo- 30.0° and finding the vector sum of the
nents of the two vectors? 60.0° 150.0 m
summed components
19. Under what circumstances would a vector have
18. They are equal and opposite.
components that are equal in magnitude?

20. The vector sum of three vectors gives a resultant Projectile Motion 19. if the vector is oriented at 45° from
equal to zero. What can you say about the vectors? REVIEWING MAIN IDEAS
the axes
27. A dart is fired horizontally from a dart gun, and
20. They form a closed triangle when
PRACTICE PROBLEMS
another dart is dropped simultaneously from the laid head to tail.
For problems 21–23, see Sample Problem A. same height. If air resistance can be neglected, which
21. a. 5 blocks at 53° north of east
dart hits the ground first?
21. A girl delivering newspapers travels three blocks west,
b. 13 blocks
four blocks north, and then six blocks east. 28. If a rock is dropped from the top of a sailboat’s mast,
a. What is her resultant displacement? will it hit the deck at the same point whether the boat 22. 42.7 yards
b. What is the total distance she travels? is at rest or in motion at constant velocity?
23. 61.8 m at 76.0° S of E (or S of W),
22. A quarterback takes the ball from the line of 29. Does a ball dropped out of the window of a moving 25.0 m at 53.1° S of E (or S of W)
scrimmage, runs backward for 10.0 yards, and then car take longer to reach the ground than one dropped
runs sideways parallel to the line of scrimmage for at the same height from a car at rest? 24. 108 m, −19.1 m
15.0 yards. At this point, he throws a 50.0-yard
forward pass straight down the field. What is the 30. A rock is dropped at the same instant that a ball at the 25. 2.81 km east, 1.31 km north
same elevation is thrown horizontally. Which will
magnitude of the football’s resultant displacement? 26. 2.40 × 102 m at 57.2° south of west
have the greater speed when it reaches ground level?
23. A shopper pushes a cart 40.0 m south down one aisle 27. Both hit at the same time.
and then turns 90.0° and moves 15.0 m. He then PRACTICE PROBLEMS
makes another 90.0° turn and moves 20.0 m. Find the 28. yes, neglecting air resistance
shopper’s total displacement. (There could be more For problems 31–33, see Sample Problem D. 29. no, neglecting air resistance
than one correct answer.)
31. The fastest recorded pitch in Major League Baseball 30. The vertical components of each
For problems 24–25, see Sample Problem B. was thrown by Nolan Ryan in 1974. If this pitch were
thrown horizontally, the ball would fall 0.809 m velocity vector will be the same, but
24. A submarine dives 110.0 m at an angle of 10.0° below (2.65 ft) by the time it reached home plate, 18.3 m the thrown ball will also have a
the horizontal. What are the two components? (60 ft) away. How fast was Ryan’s pitch?
horizontal component of velocity.
As a result, the thrown ball will have
a greater speed.
Chapter Review 109
31. 45.1 m/s

Untitled-237 109 5/4/2011 2:48:52 PM

Two-Dimensional Motion and Vectors  109


C HAPTER RE V I E W CHAPTER REVIEW

32. A person standing at the 37. A ship maneuvers to within 2.50 × 103 m of an
3 2. 3.3 s; 36 m/s edge of a seaside cliff kicks a
= +18 m/s
island’s 1.80 × 103 m high mountain peak and fires a
stone over the edge with a projectile at an enemy ship 6.10 × 102 m on the other
33. 11 m speed of 18 m/s. The cliff is side of the peak, as illustrated below. If the ship
g
34. a. 2.77 × 105 m 52 m above the water’s = 52m shoots the projectile with an initial velocity of
surface, as shown at right. 2.50 × 102 m/s at an angle of 75.0°, how close to the
b. 284 s How long does it take for enemy ship does the projectile land? How close
35. a. clears the goal by 1 m the stone to fall to the (vertically) does the projectile come to the peak?
water? With what speed does it strike the water?
b. falling 2.50 ×102 m/s
33. A spy in a speed boat is being chased down a river by
36. 4.11 m government officials in a faster craft. Just as the 1.80 ×10 3 m
75.0°
37. 80 m; 210 m officials’ boat pulls up next to the spy’s boat, both
boats reach the edge of a 5.0 m waterfall. If the spy’s 2.50 × 103 m
38. Displacement and velocity depend speed is 15 m/s and the officials’ speed is 26 m/s, how 6.10 × 102 m
on the frame of reference in which far apart will the two vessels be when they land below
the waterfall?
they are measured.
For problems 34 –37, see Sample Problem E. Relative Motion
39. the coordinate system used to REVIEWING MAIN IDEAS
34. A shell is fired from the ground with an initial speed
describe the motion
of 1.70 × 103 m/s (approximately five times the speed 38. Explain the statement “All motion is relative.”
40. Earth of sound) at an initial angle of 55.0° to the horizontal.
Neglecting air resistance, find 39. What is a frame of reference?
41. a. 70 m/s east a. the shell’s horizontal range
40. When we describe motion, what is a common frame
b. 20 m/s b. the amount of time the shell is in motion
of reference?
42. a. To the passenger, the ball appears 35. A place kicker must kick a football from a point
41. A small airplane is flying at 50 m/s toward the east.
36.0 m (about 40.0 yd) from the goal. As a result of the
to move in a straight line. To an kick, the ball must clear the crossbar, which is 3.05 m
A wind of 20 m/s toward the east suddenly begins to
blow and gives the plane a velocity of 70 m/s east.
outside observer, the ball moves high. When kicked, the ball leaves the ground with a
a. Which vector is the resultant vector?
along a parabolic trajectory. speed of 20.0 m/s at an angle of 53° to the horizontal.
b. What is the magnitude of the wind velocity?
a. By how much does the ball clear or fall short of
b. The passenger would see the ball clearing the crossbar? 42. A ball is thrown upward in the air by a passenger on a
move backward, while the b. Does the ball approach the crossbar while still train that is moving with constant velocity.
rising or while falling? a. Describe the path of the ball as seen by the
stationary observer would see no passenger. Describe the path as seen by a
change from part (a). 36. When a water gun is fired while being held horizontally
stationary observer outside the train.
at a height of 1.00 m above ground level, the water
b. How would these observations change if the train
travels a horizontal distance of 5.00 m. A child, who is
were accelerating along the track?
holding the same gun in a horizontal position, is also
sliding down a 45.0° incline at a constant speed of
2.00 m/s. If the child fires the gun when it is 1.00 m PRACTICE PROBLEMS
above the ground and the water takes 0.329 s to reach For problems 43–46, see Sample Problem F.
the ground, how far will the water travel horizontally?
43. A river flows due east at 1.50 m/s. A boat crosses the
river from the south shore to the north shore by
maintaining a constant velocity of 10.0 m/s due north
relative to the water.
a. What is the velocity of the boat as viewed by an
observer on shore?
b. If the river is 325 m wide, how far downstream is
the boat when it reaches the north shore?

110 Chapter 3

Untitled-237 110 5/4/2011 2:48:53 PM

110  Chapter 3
CHAPTER REVIEW C HAPTER RE V I E W
44. The pilot of an aircraft wishes to fly due west in a 50. An escalator is 20.0 m long. If a person stands on the
50.0 km/h wind blowing toward the south. The speed escalator, it takes 50.0 s to ride to the top. 3. a. 10.1 m/s at 8.53° east of north
4
of the aircraft in the absence of a wind is 205 km/h. a. If a person walks up the moving escalator with a
a. In what direction should the aircraft head? speed of 0.500 m/s relative to the escalator, how
b. 48.8 m
b. What should its speed relative to the ground be? long does it take the person to get to the top? 44. a. 14.1° north of west
b. If a person walks down the “up” escalator with the
45. A hunter wishes to cross a river that is 1.5 km wide same relative speed as in item (a), how long does it b. 199 km/h
and that flows with a speed of 5.0 km/h. The hunter
uses a small powerboat that moves at a maximum
take to reach the bottom? 45. 7.5 min
speed of 12 km/h with respect to the water. What is 51. A ball is projected horizontally from the edge of a 46. a. 23.2° upstream from straight
the minimum time necessary for crossing? table that is 1.00 m high, and it strikes the floor at a
point 1.20 m from the base of the table.
across
46. A swimmer can swim in still water at a speed of
9.50 m/s. He intends to swim directly across a river
a. What is the initial speed of the ball? b. 8.72 m/s across the river
b. How high is the ball above the floor when its
that has a downstream current of 3.75 m/s. velocity vector makes a 45.0° angle with the 47. a. 41.7 m/s
a. What must the swimmer’s direction be? horizontal?
b. What is his velocity relative to the bank? b. 3.81 s
52. How long does it take an automobile traveling c. vy, f = −13.5 m/s,
60.0 km/h to become even with a car that is traveling
Mixed Review in another lane at 40.0 km/h if the cars’ front vx, f = 34.2 m/s,
bumpers are initially 125 m apart? vf = 36.7 m/s
47. A ball player hits a home run, and the baseball just
clears a wall 21.0 m high located 130.0 m from 53. The eye of a hurricane passes over Grand Bahama
8. a. 15 m/s
4
home plate. The ball is hit at an angle of 35.0° to the Island. It is moving in a direction 60.0° north of west
horizontal, and air resistance is negligible. Assume with a speed of 41.0 km/h. Exactly three hours later, b. 15 m/s
the ball is hit at a height of 1.0 m above the ground. the course of the hurricane shifts due north, and its
a. What is the initial speed of the ball? speed slows to 25.0 km/h, as shown below. How far
49. 10.5 m/s
b. How much time does it take for the ball to reach from Grand Bahama is the hurricane 4.50 h after it 50. a. 22.2 s
the wall? passes over the island?
c. Find the components of the velocity and the speed b. 2.00 × 102 s
of the ball when it reaches the wall. N 51. a. 2.66 m/s
48. A daredevil jumps a canyon 12 m wide. To do so, he W E b. 0.64 m
drives a car up a 15° incline. 25.0 km/h
a. What minimum speed must he achieve to clear the S 52. 22.5 s
canyon? 53. 157 km
b. If the daredevil jumps at this minimum speed, 41.0 km/h
what will his speed be when he reaches the 60.0° 54. 7.0 × 101 s
other side?

49. A 2.00 m tall basketball player attempts a goal 10.00 m


from the basket (3.05 m high). If he shoots the ball at a
45.0° angle, at what initial speed must he throw the 54. A boat moves through a river at 7.5 m/s relative to the
basketball so that it goes through the hoop without water, regardless of the boat’s direction. If the water in
striking the backboard? PHYSICS
the river is flowing at 1.5 m/s, how long does it takeSpec. Number PH 99 PE C03-CHR-007-A
the boat to make a roundtrip consisting of a 250 mBoston Graphics, Inc.
displacement downstream followed by a 250 m 617.523.1333
displacement upstream?

Chapter Review 111

Untitled-237 111 5/4/2011 2:48:53 PM

Two-Dimensional Motion and Vectors  111


C HAPTER RE V I E W CHAPTER REVIEW

55. A car is parked on a cliff overlooking the ocean on an 60. A science student riding on a flatcar of a train moving
5 5. a. 32.5 m incline that makes an angle of 24.0° below the at a constant speed of 10.0 m/s throws a ball toward
horizontal. The negligent driver leaves the car in the caboose along a path that the student judges as
b. 1.78 s neutral, and the emergency brakes are defective. The making an initial angle of 60.0° with the horizontal.
56. a. 5.0 × 101 m/s car rolls from rest down the incline with a constant The teacher, who is standing on the ground nearby,
acceleration of 4.00 m/s2 and travels 50.0 m to the observes the ball rising vertically. How high does the
b. 4.0 × 101 m edge of the cliff. The cliff is 30.0 m above the ocean. ball rise?
57. a. 57.7 km/h at 60.0° west of the a. What is the car’s position relative to the base of the
cliff when the car lands in the ocean? 61. A football is thrown directly toward a receiver with an
vertical b. How long is the car in the air? initial speed of 18.0 m/s at an angle of 35.0° above the
horizontal. At that instant, the receiver is 18.0 m from
b. 28.8 km/h straight down 56. A golf ball with an initial angle of 34° lands exactly the quarterback. In what direction and with what
58. 12.0 s 240 m down the range on a level course. constant speed should the receiver run to catch the
a. Neglecting air friction, what initial speed would football at the level at which it was thrown?
59. 18 m; 7.9 m achieve this result?
b. Using the speed determined in item (a), find the 62. A rocket is launched at an angle of 53° above the
60. 15.3 m horizontal with an initial speed of 75 m/s, as shown
maximum height reached by the ball.
61. 6.19 m/s downfield below. It moves for 25 s along its initial line of motion
57. A car travels due east with a speed of 50.0 km/h. with an acceleration of 25 m/s2. At this time, its
62. a. 2.4 × 104 m Rain is falling vertically with respect to Earth. The engines fail and the rocket proceeds to move as a
traces of the rain on the side windows of the car make free body.
b. 152 s
an angle of 60.0° with the vertical. Find the velocity of a. What is the rocket’s maximum altitude?
c. 6.0 × 104 m the rain with respect to the following: b. What is the rocket’s total time of flight?
a. the car c. What is the rocket’s horizontal range?
b. Earth

58. A shopper in a department store can walk up a a = 25 m/s2


stationary (stalled) escalator in 30.0 s. If the normally
vi = 75 m/s
functioning escalator can carry the standing shopper
to the next floor in 20.0 s, how long would it take the
shopper to walk up the moving escalator? Assume the
same walking effort for the shopper whether the 53°
escalator is stalled or moving.

59. If a person can jump a horizontal distance of 3.0 m


on Earth, how far could the person jump on the
moon, where the free-fall acceleration is g/6 and PHYSICS
g = 9.81 m/s2? How far could the person jump on Spec. Number PH 99 PE C
Mars, where the acceleration due to gravity is 0.38g? Boston Graphics, Inc.
617.523.1333

112 Chapter 3

Untitled-237 112 5/4/2011 2:48:54 PM

112  Chapter 3
CHAPTER REVIEW C HAPTER RE V I E W
3. You are helping NASA engineers design a basketball
ALTERNATIVE ASSESSMENT
court for a colony on the moon. How do you anticipate
the ball’s motion compared with its motion on Earth?
Alternative
1. Work in cooperative groups to analyze a game of
chess in terms of displacement vectors. Make a model What changes will there be for the players—how they
move and how they throw the ball? What changes
Assessment Answers
chessboard, and draw arrows showing all the possible
moves for each piece as vectors made of horizontal would you recommend for the size of the court, the 1. Chess pieces can move in ways that
basket height, and other regulations in order to
and vertical components. Then have two members of
adapt the sport to the moon’s low gravity? Create a
require more than one component.
your group play the game while the others keep track
of each piece’s moves. Be prepared to demonstrate presentation or a report presenting your suggestions, Student reports should show how
how vector addition can be used to explain where a and include the physics concepts behind your several moves can be reported as a
recommendations.
piece would be after several moves. vector sum.
4. There is conflicting testimony in a court case. A police
2. Use a garden hose to investigate the laws of projectile
officer claims that his radar monitor indicated that a car
2. Student plans should be safe and
motion. Design experiments to investigate how the
angle of the hose affects the range of the water stream. was traveling at 176 km/h (110 mi/h). The driver argues include measurements of angle and
(Assume that the initial speed of water is constant and that the radar must have recorded the relative velocity range. They should find that 45° is
is determined by the pressure indicated by the faucet’s because he was only going 88 km/h (55 mi/h). Is it
possible that both are telling the truth? Could one be the best angle for maximum range
setting.) What quantities will you measure, and how
will you measure them? What variables do you need lying? Prepare scripts for expert witnesses, for both the and 90° is the best angle for
prosecution and the defense, that use physics to justify
to control? What is the shape of the water stream? maximum height.
How can you reach the maximum range? How can their positions before the jury. Create visual aids to be
you reach the highest point? Present your results to used as evidence to support the different arguments. 3. Students should recognize that balls
the rest of the class and discuss the conclusions. will stay in the air longer and that
players can jump higher. Players will
need to shoot lower and more
gently, or the court should be longer
and the basket higher.
4. Students should recognize that if the
cars were driving toward each other,
both drivers could be telling the
Two-Dimensional Motion truth.
Recall the following equation from your studies of projectiles In this activity, you will determine the maximum height and
launched at an angle. flight time of a baseball thrown vertically at various initial
∆y = (vi sin θ )∆t + _ 1 a (∆t)2 velocities.
2 y
Consider a baseball that is thrown straight up in the air. The Go online to HMDScience.com to find this graphing calculator
equation for projectile motion can be entered as Y1 on a activity.
graphing calculator.
Y1 = VX − 4.9X2
Given the initial velocity (V), your graphing calculator can
calculate the height (Y1) of the baseball versus the time interval
(X) that the ball remains in the air. Why is the factor sin θ
missing from the equation for Y1?

Chapter Review 113

Untitled-237 113 5/4/2011 2:48:54 PM

Two-Dimensional Motion and Vectors  113


S TA N D A R D S - B A S E D
ASSESSMENT
Standards-Based Assessment
ANSWERS
MULTIPLE CHOICE Use the passage below to answer questions 5–6.
1. B
1. Vector A has a magnitude of 30 units. Vector B is A motorboat heads due east at 5.0 m/s across a river that
2. H perpendicular to vector A and has a magnitude of flows toward the south at a speed of 5.0 m/s.
3. A 40 units. What would the magnitude of the resultant 5. What is the resultant velocity relative to an observer
vector A + B be?
4. J A. 10 units
on the shore?
A. 3.2 m/s to the southeast
5. C B. 50 units B. 5.0 m/s to the southeast
C. 70 units C. 7.1 m/s to the southeast
6. G D. zero D. 10.0 m/s to the southeast
7. B 2. What term represents the magnitude of a velocity 6. If the river is 125 m wide, how long does the boat
8. J vector? take to cross the river?
F. acceleration F. 39 s
G. momentum G. 25 s
H. speed H. 17 s
J. velocity J. 12 s
Use the diagram below to answer questions 3–4.
7. The pilot of a plane measures an air velocity of
165 km/h south relative to the plane. An observer
y on the ground sees the plane pass overhead at a
B velocity of 145 km/h toward the north. What is the
cm
velocity of the wind that is affecting the plane
2 . 0 relative to the observer?
30.0°
x A. 20 km/h to the north
2.0 cm A B. 20 km/h to the south
C. 165 km/h to the north
3. What is the direction of the resultant vector A + B? D. 310 km/h to the south
A. 15° above the x-axis
B. 75° above the x-axis 8. A golfer takes two putts to sink his ball in the hole
C. 15° below the x-axis once he is on the green. The first putt displaces the
D. 75° below the x-axis ball 6.00 m east, and the second putt displaces the
ball 5.40 m south. What displacement would put the
4. What is the direction of the resultant vector A – B? ball in the hole in one putt?
F. 15° above the x-axis F. 11.40 m southeast
G. 75° above the x-axis G. 8.07 m at 48.0° south of east
H. 15° below the x-axis H. 3.32 m at 42.0° south of east
J. 75° below the x-axis J. 8.07 m at 42.0° south of east

114 Chapter 3

Untitled-308 114 5/6/2011 11:58:36 AM

114  Chapter 3
TEST PREP

9. D
Use the information below to answer questions 9–12. 14. A roller coaster travels 41.1 m at an angle of 40.0° 10. H
above the horizontal. How far does it move
A girl riding a bicycle at 2.0 m/s throws a tennis ball
horizontally and vertically?
11. B
horizontally forward at a speed of 1.0 m/s from a height
of 1.5 m. At the same moment, a boy standing on the 15. A ball is thrown straight upward and returns to the
12. H
sidewalk drops a tennis ball straight down from a height thrower’s hand after 3.00 s in the air. A second ball is 13. They are perpendicular.
of 1.5 m. thrown at an angle of 30.0° with the horizontal. At
what speed must the second ball be thrown to reach 14. 31.5 m horizontally, 26.4 m vertically
9. What is the initial speed of the girl’s ball relative to
the boy?
the same height as the one thrown vertically? 15. 29.4 m/s
A. 1.0 m/s 16. 10.8 m
B. 1.5 m/s EXTENDED RESPONSE
C. 2.0 m/s 17. Executive C is correct. Student
16. A human cannonball is shot out of a cannon at 45.0°
D. 3.0 m/s explanations should include the
to the horizontal with an initial speed of 25.0 m/s. A
10. If air resistance is disregarded, which ball will hit the net is positioned at a horizontal distance of 50.0 m concept of relative velocity—when a
ground first? from the cannon. At what height above the cannon
helicopter or airship lifts off straight
F. the boy’s ball should the net be placed in order to catch the
G. the girl’s ball human cannonball? Show your work. up from the ground, it is already
H. neither moving horizontally with Earth’s
J. The answer cannot be determined from the given Use the passage below to answer question 17.
information.
horizontal velocity. (We assume that
Three airline executives are discussing ideas for devel- Earth’s motion is constant for the
11. If air resistance is disregarded, which ball will have a oping flights that are more energy efficient.
greater speed (relative to the ground) when it hits
purposes of this scenario and does
Executive A: Because the Earth rotates from west to east,
the ground? not depend on time.)
we could operate “static flights”—a helicopter or airship
A. the boy’s ball
could begin by rising straight up from New York City and
B. the girl’s ball
then descend straight down four hours later when San
C. neither
Francisco arrives below.
D. The answer cannot be determined from the given
information. Executive B: This approach could work for one-way
flights, but the return trip would take 20 hours.
12. What is the speed of the girl’s ball when it hits the
ground? Executive C: That approach will never work. Think about
F. 1.0 m/s it. When you throw a ball straight up in the air, it comes
G. 3.0 m/s straight back down to the same point.
H. 6.2 m/s
J. 8.4 m/s Executive A: The ball returns to the same point because
Earth’s motion is not significant during such a short
time.
SHORT RESPONSE
17. In a paragraph, state which of the executives is
13. If one of the components of one vector along the correct, and explain why.
direction of another vector is zero, what can you
conclude about these two vectors?

11 12 1
Test Tip
10 2
9 3 If you get stuck answering a question,
8 4 move on. You can return to the question
7 6 5
later if you have time.

Standards-Based Assessment 115

Untitled-308 115 5/6/2011 11:58:37 AM

Two-Dimensional Motion and Vectors  115

You might also like